*NURSING > EXAM > HESI Exit V2 (Nurse Hero Review) 221 Questions and Answers,100% CORRECT (All)

HESI Exit V2 (Nurse Hero Review) 221 Questions and Answers,100% CORRECT

Document Content and Description Below

HESI Exit V2 (Nurse Hero Review) 221 Questions and Answers 1. The LPN/LVN is preparing to ambulate a postoperative client after cardiac surgery. The nurse plans to do which to enable the client to... best tolerate the ambulation? 1. Provide the client with a walker. 2. Remove the telemetry equipment. 3. Encourage the client to cough and deep breathe. 4. Premedicate the client with an analgesic before ambulating. 2. A client is wearing a continuous cardiac monitor, which begins to alarm at the nurse's station. The nurse sees no electrocardiographic complexes on the screen. The nurse should do which first? a. Call a code blue. b. Call the health care provider. c. Check the client status and lead placement. d. Press the recorder button on the ECG console. 3. 3) The LPN/LVN in a medical unit is caring for a client with heart failure. The client suddenly develops extreme dyspnea, tachycardia, and lung crackles, and the nurse suspects pulmonary edema. The nurse immediately notifies the registered nurse and expects which interventions to be prescribed? Select all that apply. a. Administering oxygen b. Inserting a Foley catheter c. Administering furosemide (Lasix) d. Administering morphine sulfate intravenously e. Transporting the client to the coronary care unit f. Placing the client in a low-Fowler's side-lying position 4. The nurse is monitoring a client following cardioversion. Which observations should be of highest priority to the nurse? a. Blood pressure b. Status of airway c. Oxygen flow rate d. Level of consciousness 5. The nurse is assisting in caring for the client immediately after insertion of a permanent demand pacemaker via the right subclavian vein. The nurse prevents dislodgement of the pacing catheter by implementing which intervention? a. Limiting movement and abduction of the left arm b. Limiting movement and abduction of the right arm c. Assisting the client to get out of bed and ambulate with a walker 4. Having the physical therapist do active range of motion to the right arm 6. A client diagnosed with thrombophlebitis 1 day ago suddenly complains of chest pain and shortness of breath, and the client is visibly anxious. The LPN/LVN understands that a life-threatening complication of this condition is which? a. Pneumonia b. Pulmonary edema c. Pulmonary embolism d. Myocardial infarction 7. A 24-year-old man seeks medical attention for complaints of claudication in the arch of the foot. The nurse also notes superficial thrombophlebitis of the lower leg. The nurse should check the client for which next? a. Smoking history b. Recent exposure to allergens c. History of recent insect bites d. Familial tendency toward peripheral vascular disease 8. The nurse has reinforced instructions to the client with Raynaud's disease about self-management of the disease process. The nurse determines that the client needs further teaching if the client states which? a. "Smoking cessation is very important." b. "Moving to a warmer climate should help." c. "Sources of caffeine should be eliminated from the diet." 4. "Taking nifedipine (Procardia) as prescribed will decrease vessel spasm." 9. A client with myocardial infarction suddenly becomes tachycardic, shows signs of air hunger, and begins coughing frothy, pink- tinged sputum. The nurse listens to breath sounds, expecting to hear which breath sounds bilaterally? a. Rhonchi b. Crackles c. Wheezes d. Diminished breath sounds 10. The LPN/LVN is collecting data on a client with a diagnosis of right sided heart failure. The nurse should expect to note which specific characteristic of this condition? a. Dyspnea b. Hacking cough c. Dependent edema d. Crackles on lung auscultation 11. The LPN/LVN is checking the neurovascular status of a client who returned to the surgical nursing unit 4 hours ago after undergoing an aortoiliac bypass graft. The affected leg is warm, and the nurse notes redness and edema. The pedal pulse is palpable and unchanged from admission. The nurse interprets that the neurovascular status is which? a. Moderately impaired, and the surgeon should be called b. Normal, caused by increased blood flow through the leg c. Slightly deteriorating, and should be monitored for another hour d. Adequate from an arterial approach, but venous complications are arising 12. A client with a diagnosis of rapid rate atrial fibrillation asks the nurse why the health care provider is going to perform carotid massage. The LPN/LVN responds that this procedure may stimulate which? a. Vagus nerve to slow the heart rate b. Vagus nerve to increase the heart rate c. Diaphragmatic nerve to slow the heart rate d. Diaphragmatic nerve to increase the heart rate 13. A client is admitted to the hospital with possible rheumatic endocarditis. The LPN/LVN should check for a history of which type of infection? a. Viral infection b. Yeast infection c. Streptococcal infection d. Staphylococcal infection 14. A client has an Unna boot applied for treatment of a venous stasis leg ulcer. The LPN/LVN notes that the client's toes are mottled, and cool and the client verbalizes some numbness and tingling of the foot. Which interpretation should the nurse make of these findings? a. The boot has not yet dried. b. The boot is controlling leg edema. c. The boot is impairing venous return. d. The boot has been applied too tightly. 15. A client with angina complains that the anginal pain is prolonged and severe and occurs at the same time each day, most often in the morning. On further data collection, the nurse notes that the pain occurs in the absence of precipitating factors. How should the LPN/LVN best describe this type of anginal pain? a. Stable angina b. Variant angina c. Unstable angina d. Nonanginal pain 16. The LPN/LVN is monitoring a client with an abdominal aortic aneurysm (AAA). Which finding is probably unrelated to the AAA? a. Pulsatile abdominal mass b. Hyperactive bowel sounds in the area c. Systolic bruit over the area of the mass d. Subjective sensation of "heart beating" in the abdomen 17. An emergency department client who complains of slightly improved but unrelieved chest pain for 2 days is reluctant to take a nitroglycerin sublingual tablet offered by the nurse. The client states, "I don't need that—my dad takes that for his heart. There's nothing wrong with my heart." Which description best describes the client's response? a. Angry b. Denial c. Phobic d. Obsessive-compulsive 18. A client is scheduled for a cardiac catheterization using a radiopaque dye. The LPN/LVN checks which most critical item before the procedure? a. Intake and output b. Height and weight c. Peripheral pulse rates d. Prior reaction to contrast media 19. A client is scheduled for a dipyridamole thallium scan. The LPN/ LVN should check to make sure that the client has not consumed which substance before the procedure? a. Caffeine b. Fatty meal c. Excess sugar d. Milk products 20. An ambulatory clinic nurse is interviewing a client who is complaining of flulike symptoms. The client suddenly develops chest pain. Which question best assists the nurse to discriminate pain caused by a non-Cardiac problem? a. "Can you describe the pain to me?" b. "Have you ever had this pain before?" c. "Does the pain get worse when you breathe in?" 21. A client with myocardial infarction (MI) has been transferred from the coronary care unit (CCU) to the general medical unit with cardiac monitoring via telemetry. The nurse assisting in caring for the client expects to note which type of activity prescribed? a. Strict bed rest for 24 hours b. Bathroom privileges and self-care activities c. Unrestricted activities because the client is monitored d. Unsupervised hallway ambulation with distances less than 200 feet 22. The LPN/LVN is preparing to care for a client who will be arriving from the recovery room after an above-the-knee amputation. The nurse ensures that which priority item is available for emergency use? a. Surgical tourniquet b. Dry sterile dressings c. Incentive spirometer d. Over-the-bed trapeze 23. A client is diagnosed with thrombophlebitis. The nurse should tell the client that which prescription is indicated? a. Bed rest, with bathroom privileges only b. Bed rest, keeping the affected extremity flat c. Bed rest, with elevation of the affected extremity d. Bed rest, with the affected extremity in a dependent position 24. A client returns to the nursing unit after an above knee amputation of the right leg. In which position should the nurse place the client? a. Prone with the head on a pillow b. With the foot of the bed elevated c. Reverse Trendelenburg's position d. With the residual limb flat on the bed 25. The LPN/LVN is collecting data from a client about medications being taken, and the client tells the nurse that he is taking herbal supplements for the treatment of varicose veins. The nurse understands that the client is most likely taking which? a. Bilberry b. Ginseng c. Feverfew d. Evening primrose 25. The LPN/LVN is planning to reinforce instructions to a client with peripheral arterial disease about measures to limit disease progression. The nurse should include which items on a list of suggestions to be given to the client? Select all that apply. a. Wear elastic stockings. b. Be careful not to injure the legs or feet. c. Use a heating pad on the legs to aid vasodilation. d. Walk each day to increase circulation to the legs. e. Cut down on the amount of fats consumed in the diet. 27. A client is at risk for developing disseminated intravascular coagulopathy (DIC). The LPN/LVN should become concerned with which fibrinogen level? a. 90 mg/dL b. 190 mg/dL c. 290 mg/dL d. 390 mg/dL 28. A hospitalized client with a history of angina pectoris is ambulating in the corridor. The client suddenly complains of severe substernal chest pain. The LPN/LVN should take which action first? a. Check the client's vital signs. b. Assist the client to sit or lie down. c. Administer sublingual nitroglycerin. d. Apply nasal oxygen at a rate of 2 L/min. 29. The LPN/LVN notes bilateral 2+ edema in the lower extremities of a client with known coronary artery disease who was admitted to the hospital 2 days ago. Based on this finding, the nurse should implement which action? a. Reviews the intake and output records for the last 2 days b. Prescribes daily weights starting on the following morning c. Changes the time of diuretic administration from morning to evening d. Requests a sodium restriction of 1 g/day from the health care provider 30. A client brings the following medications to the clinic for a yearly physical. The LPN/LVN realizes which medication has been prescribed to treat heart failure? a. Digoxin (Lanoxin) b. Warfarin (Coumadin) c. Amiodarone (Cordarone) d. Potassium chloride (K-Dur) 31. A student nurse is assigned to assist in caring for a client with acute pulmonary edema who is receiving digoxin (Lanoxin) and heparin therapy. The nursing instructor reviews the plan of care formulated by the student and tells the student that which intervention is unsafe? a. Restricting the client's potassium intake b. Encouraging the client to rest after meals c. Administering the heparin with a 25-gauge needle d. Holding the digoxin for a heart rate less than 60 beats per minute 32. A client has an inoperable abdominal aortic aneurysm (AAA). Which measure should the nurse anticipate reinforcing when teaching the client? a. Bed rest b. Restricting fluids c. Antihypertensives d. Maintaining a low-fiber diet 33. The LPN/LVN finds a client tensing while lying in bed staring at the cardiac monitor. Which is the nurse's best response when the client states, "There sure are a lot of wires around there. I sure hope we don't get hit by lightning!"? a. "Would you like a mild sedative to help you relax?" b. "Oh, don't worry, the weather is supposed to be sunny and clear today." c. "Yes, this equipment is a little scary. Can we talk about how the cardiac monitor works?" d. "I can appreciate your concerns. Your family can stay with you tonight if you want them to." 34. The LPN/LVN is asked to assist another health care member in providing care to a client who is placed in a modified Trendelenburg's position. The nurse interprets that the client is likely being treated for which condition? a. Shock b. Kidney dysfunction c. Respiratory insufficiency d. Increased intracranial pressure 35. A client is seen in the health care provider's office for a physical examination after experiencing unusual fatigue over the last several weeks. Height is 5 feet, 8 inches, with a weight of 220 pounds. Vital signs are temperature 98.6° F oral, pulse 86 beats per minute, respirations 18 breaths per minute, and blood pressure 184/96 mm Hg. Random blood glucose is 110 mg/dL. In order to best collect relevant data, which question should the LPN/LVN ask the client first? a. "Do you exercise regularly?" b. "Would you consider losing weight?" c. "Is there a history of diabetes mellitus in your family?" d. "When was the last time you had your blood pressure checked?" 36. The client scheduled for a right femoropopliteal bypass graft is at risk for compromised tissue perfusion to the extremity. The LPN/LVN takes which action before surgery to address this risk? a. Having the client void before surgery b. Completing a preoperative checklist c. Marking the location of the pedal pulses on the right leg d. Checking the results of any baseline coagulation studies 37. When preparing a client for a pericardiocentesis, which position does the LPN/LVN place the client in? a. Supine with slight lowering of the head b. Lying on the right side with a pillow under the head c. Lying on the left side with a pillow under the chest wall d. Supine with the head of bed elevated at a 45- to 60-degree angle 38. For a client diagnosed with pulmonary edema, the LPN/LVN establishes a goal to have the client participate in activities that reduce cardiac workload. Which client activities will contribute to achieving this goal? a. Elevating the legs when in bed b. Sleeping in the supine position c. Using a bedside commode for stools d. Seasoning beef with a meat tenderizer 39. The LPN/LVN is caring for a client who is developing pulmonary edema. The client exhibits respiratory distress, but the blood pressure is unchanged from the client's baseline. As an immediate action before help arrives, the nurse should perform which action? a. Suction the client vigorously. b. Place the client in high-Fowler's position. c. Begin assembling medications that are anticipated to be given. d. Call the respiratory therapy department to request a ventilator. 40. The LPN/LVN has reinforced home care instructions to a client who had a permanent pacemaker inserted. Which educational outcome has the greatest impact on the client's long-term cardiac health? a. Knowledge of when it is safe to resume sexual activity b. The ability to take an accurate pulse in either the wrist or neck c. An understanding of the importance of proper microwave oven usage d. An understanding of why vigorous arm and shoulder movement must be avoided initially 41. The clinic nurse is obtaining cardiovascular data on a client. The LPN/LVN prepares to check the client's apical pulse and places the stethoscope in which position? a. Midsternum equal with the nipple line b. At the midaxillary line on the left side of the chest c. At the midline of the chest just below the xiphoid process d. At the midclavicular line at the fifth left intercostal space 42. The LPN/LVN is caring for a client who has been admitted to the hospital with a diagnosis of angina pectoris. The client is receiving oxygen via nasal cannula at 2 L. The client asks the nurse why the oxygen is necessary. The LPN/LVN bases the response on which information? a. Oxygen assists in calming the client. b. Oxygen prevents the development of any thrombus formation. c. Deficient oxygenation to heart cells results in angina pectoris pain. d. Oxygen dilates the blood vessels, supplying more nutrients to the heart muscle. 43. The licensed practical nurse (LPN) is assisting in caring for a client with a diagnosis of myocardial infarction (MI). The client is experiencing chest pain that is unrelieved by the administration of nitroglycerin. The registered nurse administers morphine sulfate to the client as prescribed by the health care provider. Following administration of the morphine sulfate, the LPN plans to monitor which indicator(s)? a. Mental status b. Urinary output c. Respirations and blood pressure d. Temperature and blood pressure 44. A client diagnosed with angina pectoris returns to the nursing unit after experiencing an angioplasty. The nurse reinforces instructions to the client regarding the procedure and home care measures. Which statement by the client indicates an understanding of the instructions? a. "I am considering cutting my workload." b. "I need to cut down on cigarette smoking." c. "I am so relieved that my heart is repaired." d. "I need to adhere to my dietary restrictions." 45. The LPN/LVN is caring for a client with a diagnosis of myocardial infarction (MI) and is assisting the client in completing the diet menu. Which beverage does the nurse instruct the client to select from the menu? a. Tea b. Cola c. Coffee d. Lemonade 45. 46) The LPN/LVN is collecting data on a client with a diagnosis of angina pectoris who takes nitroglycerin for chest pain. During the admission, the client reports chest pain. The nurse immediately asks the client which question? a. "Are you having any nausea?" b. "Where is the pain located?" c. "Are you allergic to any medications?" d. "Do you have your nitroglycerin with you?" 47. The LPN/LVN has reinforced dietary instructions to a client with coronary artery disease. Which statement by the client indicates an understanding of the dietary instructions? a. "I need to substitute eggs and milk for meat." b. "I will eliminate all cholesterol and fat from my diet." c. "I should routinely use polyunsaturated oils in my diet." d. "I need to seriously consider becoming a strict vegetarian." 47. The LPN/LVN is assisting in caring for a client in the telemetry unit who is receiving an intravenous infusion of 1000 mL 5% dextrose with 40 mEq of potassium chloride. Which occurrence observed on the cardiac monitor indicates the presence of hyperkalemia? a. Tall, peaked T waves b. ST segment depressions c. Shortened P-R intervals d. Shortening of the QRS complex 49. The LPN/LVN is assisting in caring for a client in the telemetry unit and is monitoring the client for cardiac changes indicative of hypokalemia. Which occurrence noted on the cardiac monitor indicates the presence of hypokalemia? a. Tall, peaked T waves b. ST-segment depression c. Prolonged P-R interval d. Widening of the QRS complex 50. While the nurse is involved in preparing a client for a cardiac catheterization, the client says, "I don't want to talk with you. You're only the nurse. I want my doctor." Which response by the nurse should be therapeutic? a. "Your doctor expects me to prepare you for this procedure." b. "That's fine, if that's what you want. I'll call your health care provider." c. "So you're saying that you want to talk to your health care provider?" d. "I'm concerned with the way you've dismissed me. I know what I am doing." 51. The LPN/LVN reinforces instructions to a client at risk for thrombophlebitis regarding measures to minimize its occurrence. Which statement by the client indicates an understanding of this information? a. "I need to avoid pregnancy by taking oral contraceptives." b. "I should avoid sitting in one position for long periods of time." c. "I can finally stop wearing these support stockings that you gave me." d. "I will be sure to maintain my fluid intake to at least four glasses daily." 52. A client with a history of angina pectoris tells the nurse that chest pain usually occurs after going up two flights of stairs or after walking four blocks. The LPN/LVN interprets that the client is experiencing which type of angina? a. Stable b. Variant c. Unstable d. Intractable 53. The LPN/LVN is teaching the client with angina pectoris about disease management and lifestyle changes that are necessary in order to control disease progression. Which statement by the client indicates a need for further teaching? a. "I will avoid using table salt with meals." b. "It is best to exercise once a week for an hour." c. "I will take nitroglycerin whenever chest discomfort begins." d. "I will use muscle relaxation to cope with stressful situations." 54) The LPN/LVN is working with a client who has been diagnosed with Prinzmetal's (variant) angina. The nurse plans to reinforce which information about this type of angina when teaching the client? a. Prinzmetal's angina is effectively managed by beta-blocking agents. b. Prinzmetal's angina improves with a low-sodium, high-potassium diet. c. Prinzmetal's angina has the same risk factors as stable and unstable angina. d. Prinzmetal's angina is generally treated with calcium channel blocking agents. 55. The LPN/LVN working in a long-term care facility is collecting data from a client experiencing chest pain. The nurse should interpret that the pain is likely a result of myocardial infarction (MI) if which observation is made by the nurse? a. The client is not experiencing nausea or vomiting. b. The pain is described as substernal and radiating to the left arm. c. The pain has not been unrelieved by rest and nitroglycerin tablets. d. The client says the pain began while trying to open a stuck dresser drawer. 56. The LPN/LVN is discussing smoking cessation with a client diagnosed with coronary artery disease (CAD). Which statement should the nurse make to the client to try to motivate the client to quit smoking? a. "Since the damage has already been done, it will be all right to cut down a little at a time." b. "None of the cardiovascular effects are reversible, but quitting might prevent lung cancer." c. "If you totally quit smoking right now, you can cut your cardiovascular risk to zero within a year." d. "If you quit now, your risk of cardiovascular disease will decrease to that of a nonsmoker in 3 to 4 years." 57. A client with heart failure is scheduled to be discharged to home with digoxin (Lanoxin) and furosemide (Lasix) as ongoing prescribed medications. The nurse teaches the client to report which sign/symptom that indicates the medications are not producing the intended effect? a. Decrease in pedal edema b. High urine output during the day c. Weight gain of 2 to 3 pounds in a few days d. Cough accompanied by other signs of respiratory infection 58. A client has experienced an episode of pulmonary edema. The LPN/ LVN determines that the client's respiratory status is improving if which breath sounds are noted? a. Rhonchi b. Wheezes c. Crackles in the lung bases d. Crackles throughout the lung fields 59. A client in pulmonary edema has a prescription to receive morphine sulfate intravenously. The licensed practical nurse assisting in caring for the client determines that the client experienced an intended effect of the medication if which is noted? a. Increased pulse rate b. Relief of apprehension c. Decreased urine output d. Increased blood pressure 60. The LPN/LVN is providing discharge teaching for a post- myocardial infarction (MI) client who will be taking 1 baby aspirin a day. The nurse determines that the client understands the use of this medication if the client makes which statement? a. "I will take this medication every day." b. "I will take this medication every other day." c. "I will take this medication until I feel better." d. "I will take this medication only when I have pain." 61. The LPN/LVN determines that a client with coronary artery disease (CAD) needs further teaching about disease management if the client makes which statement? a. "I will watch my weight gain." b. "I will avoid walking for exercise." c. "I will monitor my cholesterol intake." d. "I will follow a low-fat, low-salt diet." 62. An older client with ischemic heart disease has experienced an episode of dizziness and shortness of breath. The nurse reviews the plan of care and notices documentation of decreased cardiac output, dyspnea, and syncopal episodes. The nurse plans to take which important action? a. Monitor oxygen saturation levels. b. Place the client on a cardiac monitor. c. Measure blood pressure every 4 hours. d. Check capillary refill at least once per shift. 63. The LPN/LVN is planning adaptations needed for activities of daily living for a client with cardiac disease. The nurse should incorporate which instruction in discussion with the client? a. Increase fluids to 3000 mL per day to promote renal perfusion. b. Consume 1 to 2 oz of liquor each night to promote vasodilation. c. Try to engage in vigorous activity to strengthen cardiac reserve. d. Take in adequate daily fiber to prevent straining during a bowel movement. 64. An adult client just admitted to the hospital with heart failure also has a history of diabetes mellitus. The nurse calls the health care provider to verify a prescription for which medication that the client was taking before admission? a. NPH insulin b. Regular insulin c. Chlorpropamide d. Acarbose (Precose) 65. Acetylsalicylic acid (aspirin) is prescribed for a client before a percutaneous transluminal coronary angioplasty (PTCA). When the nurse takes the aspirin to the client, the client asks the nurse about its purpose. What is the purpose of the aspirin? a. To prevent the formation of clots b. To relieve pain at the injection site c. To prevent a fever after the procedure d. To prevent inflammation of the injection site 66. The nurse is caring for a client with coronary artery disease, and a topical nitrate is prescribed for the client. Why is acetaminophen (Tylenol) usually prescribed to be taken before the administration of the topical nitrate? a. Headache is a common side effect of nitrates. b. Fever usually accompanies coronary artery disease. c. Acetaminophen potentiates the therapeutic effects of nitrates. d. Acetaminophen does not interfere with platelet action as acetylsalicylic acid (aspirin) does. 67. The nurse is assisting in developing a plan of care for a client who will be returning to the nursing unit following a cardiac catheterization via the femoral approach. Which nursing intervention should be included in the post procedure plan of care? a. Place the client's bed in the Fowler's position. b. Encourage the client to increase fluid intake. c. Instruct the client to perform range-of-motion exercises of the extremities. d. Hold regularly scheduled medications for 24 hours following the procedure. 68. The nurse is reinforcing dietary instructions to a client with heart failure (HF). The nurse determines that the client understands the instructions if the client states that which food item will be avoided? a. Catsup b. Sherbet c. Cooked cereal d. Leafy green vegetables 69. A client seeks medical attention for intermittent episodes in which the fingers of both hands become cold, pale, and numb. The client states that they then become reddened and swollen with a throbbing, achy pain and Raynaud's disease is diagnosed. Which factor would precipitate these episodes? a. Exposure to heat b. Being in a relaxed environment c. Prolonged episodes of inactivity d. Ingestion of coffee or chocolate 70. A client is admitted to the hospital with a diagnosis of pericarditis. The nurse reviews the client's record for which sign or symptom that differentiates pericarditis from other cardiopulmonary problems? a. Anterior chest pain b. Pericardial friction rub c. Weakness and irritability d. Chest pain that worsens on inspiration 71. The nurse is beginning to ambulate a client with activity intolerance caused by bacterial endocarditis. The nurse determines that the client is best tolerating ambulation if which parameter is noted? a. Mild dyspnea after walking 10 feet b. Minimal chest pain rated 1 on a 1-to-10 pain scale c. Pulse rate that increases from 68 to 94 beats per minute d. Blood pressure that increases from 114/82 to 118/86 mm Hg 72. The nurse is assisting a hospitalized client who is newly diagnosed with coronary artery disease (CAD) to make appropriate selections from the dietary menu. The nurse encourages the client to select which meal? a. Sausage, pancakes, and toast b. Broccoli, buttered rice, and grilled chicken c. Hamburger, baked apples, and avocado salad d. Fresh strawberries, steamed vegetables, and baked fish 73. A client with known coronary artery disease (CAD) begins to experience chest pain while getting out of bed. The nurse should take which action? a. Get a prescription for pain medication. b. Have the client stop and lie back down in bed. c. Report the complaint to the health care provider. d. Have the client continue to get out of bed and into a chair. 74. The nurse is setting up the bedside unit for a client being admitted to the nursing unit from the emergency department with a diagnosis of coronary artery disease (CAD). The nurse should place highest priority on making sure that which is available at the bedside? a. Bedside commode b. Rolling shower chair c. Oxygen tubing and flowmeter d. Twelve-lead electrocardiogram (ECG) machine 75. The nurse determines that a client with coronary artery disease (CAD) understands disease management if the client makes which statement? a. "I will walk for one-half hour daily." b. "As long as I exercise I can eat anything I wish." c. "My weight has nothing to do with this disease." d. "It doesn't matter if my father had high cholesterol." 76. A client has just completed an information session about measures to minimize the progression of coronary artery disease (CAD). Which statement indicates an initial understanding of lifestyle alterations? a. I should take daily medication for life. b. I should eat a diet that is low in fat and cholesterol. c. I should continue to smoke to keep the metabolic rate high. d. I should begin to exercise if diet is not sufficient to achieve weight loss. 77. The nurse is collecting data on a client who was just admitted to the hospital with a diagnosis of coronary artery disease (CAD). The client reveals having been under a great deal of stress recently. Which should the nurse do next? a. Ask whether the client wants to see a psychiatrist. b. Explore with the client the sources of stress in life. c. Reassure the client that everybody seems stressed these days. d. Ask the client to write down a list of stressors to be evaluated at a later time. 78. A client with a diagnosis of myocardial infarction has a new activity prescription allowing the client to have bathroom privileges. As the client stands and begins to walk, the client begins to complain of chest pain. The nurse should take which action? a. Assist the client to get back into bed. b. Report the chest pain episode to the health care provider. c. Tell the client to stand still, and take the client's blood pressure. d. Give a nitroglycerin (Nitrostat) tablet, and assist the client to the bathroom. 79. A client being seen in the emergency department for complaints of chest pain confides in the nurse about regular use of cocaine as a recreational drug. The nurse takes which important action in delivering holistic nursing care to this client? a. Reports the client to the police for illegal drug use b. Explains to the client the damage that cocaine does to the heart c. Tells the client it is imperative to stop before myocardial infarction occurs d. Teaches about the effects of cocaine on the heart and offers referral for further help 80. The nurse is planning measures to decrease the incidence of chest pain for a client with angina pectoris. The nurse should do which intervention to effectively accomplish this goal? a. Provide a quiet and low-stimulus environment. b. Encourage the family to come visit very frequently. c. Encourage the client to call friends and relatives each day. d. Recommend that the client watch TV as a constant diversion. 81. A client in a long-term care facility who has a history of angina pectoris wants to go for a short walk outside with a family member. It is a sunny but chilly December day. The nurse should perform which intervention to care for this client in a holistic manner? a. Tell the client that this is not allowed. b. Tell the family member not to take the client outdoors. c. Give the client a cup of hot coffee before going outside. d. Instruct the family member to dress the client warmly before going outside. 82. The LPN/LVN carries out a standard prescription for a stat electrocardiogram (ECG) on a client who has an episode of chest pain. The nurse should take which action next? a. Do a repeat 12-lead ECG. b. Wait to see whether the pain resolves. c. Report the episode of chest pain to the health care provider. d. Give sublingual nitroglycerin (Nitrostat) per the health care provider's prescriptions. 83. A client admitted to the hospital with a diagnosis of myocardial infarction (MI) tells the nurse that the pain likely resulted from the fried chicken sandwich that the client had for lunch. The nurse's response is based on which fact? a. Most people love high-fat diets. b. Denial is a common occurrence early after MI. c. The client probably wants to belittle the opinion of the staff. d. The client is not motivated to learn about heart disease at this time. 84. The nurse is preparing to provide a therapeutic environment for a client who recently had a myocardial infarction (MI). Which are characteristics of a therapeutic environment? a. No stimulus, no stress b. Low stimulus, low stress c. High stimulus, low stress d. Moderate stimulus, low stress 85. A client who experienced a myocardial infarction (MI) tells the nurse that he is fearful about not being able to return to a normal life. Which action by the nurse is therapeutic at this time? a. Tell the client that his fears are not rational. b. Tell the client that his life has not changed. c. Explore the specific concerns with the client. d. Tell the client to talk it out with the significant other. 86. A client complaining of chest pain has an as-needed (PRN) prescription for sublingual nitroglycerin (Nitrostat). Before administering the medication to the client, the nurse should first check which? a. Blood pressure b. Cardiac rhythm c. Respiratory rate d. Peripheral pulses 87. A client who has undergone femoropopliteal bypass grafting says to the nurse, "I hope I don't have any more problems that could make me lose my leg. I'm so afraid that I'll have gone through this for nothing." Which is an appropriate nursing response? a. "There is nothing to worry about." b. "You are concerned about losing your leg?" c. "There are many people with the same problem, and they are doing just fine." d. "You have the best health care provider in the city, and your health care provider will not let anything happen to you." 88. The nurse is teaching a hospitalized client who has had aortoiliac bypass grafting about measures to improve circulation. The nurse should tell the client to do which? a. Bend the leg at the hip. b. Keep the ankles uncrossed. c. Place two pillows under the knees. d. Use the knee gatch on the bed controls. 89. A client is admitted to the hospital with possible rheumatic heart disease. The LPN/LVN collects data from the client and checks the client for which signs/symptoms? a. Skin scratches b. Vaginal itching c. Fever and sore throat d. Burning on urination 90. A client with infective endocarditis is at risk for heart failure. The nurse monitors the client for which signs and symptoms of heart failure? a. Lung crackles, peripheral edema, and weight gain b. Confusion, decreasing level of consciousness, and aphasia c. Respiratory distress, chest pain, and the use of accessory muscles d. Flank pain with radiation to the groin, accompanied by hematuria 91. A client has just returned from the cardiac catheterization laboratory. The left femoral vessel was used as the access site. After returning the client to bed and conducting an initial assessment, the nurse assisting in caring for the client expects the health care provider to write a prescription for the client to remain on bed rest. In which position should the bed be positioned? a. In the high-Fowler's position b. With the head of bed elevated at least 60 degrees c. With the head of bed elevated no more than 30 degrees d. With the foot of bed elevated as much as tolerated by the client Correct 92. The nurse is collecting data from a client with varicose veins. Which finding would the nurse identify as an indication of a potential complication associated with this disorder? a. Legs are unsightly in appearance and distress the client. b. The client complains of aching and feelings of heaviness in the legs. c. The client complains of leg edema, and skin breakdown has started. d. The health care provider finds that the legs become distended when the tourniquet is released during the Trendelenburg's test. 93. A client with coronary artery disease has selected guided imagery to help cope with psychological stress. Which statement by the client indicates understanding of this stress reduction measure? a. "This will help only if I play music at the same time." b. "This will work for me only if I am alone in a quiet area." c. "I need to do this only when I lie down in case I fall asleep." d. "The best thing about this is that I can use it anywhere, anytime." 94. A client, who is 36 hours’ post-myocardial infarction, has ambulated for the first time. The nurse determines that the client best tolerated the activity if which observation is made? a. The skin is cool but slightly diaphoretic. b. Dyspnea is noted only at the end of the exercise. c. The pre activity pulse rate is 86 beats per minute; the post activity pulse rate is 94 beats per minute. d. The pre activity blood pressure (BP) is 140/84 mm Hg; the post activity BP is 110/72 mm Hg. 95. The nurse is planning a dietary menu for a client with heart failure being treated with digoxin (Lanoxin) and furosemide (Lasix). Which would be the best dinner choice from the daily menu? a. Beef ravioli, spinach soufflé, and Italian bread b. Baked pollock, mashed potatoes, and carrot-raisin salad c. Roasted chicken breast, brown rice, and stewed tomatoes d. Beef vegetable soup, macaroni and cheese, and a dinner roll 96. A client has received instructions about an upcoming cardiac catheterization. The nurse determines that the client has the best understanding of the procedure if the client knows to report which symptoms? a. Chest pain b. Urge to cough c. Warm, flushed feeling d. Pressure at the insertion site 97. The nurse is caring for a client diagnosed with Buerger's disease. Which finding should the nurse determine is a potential complication associated with this disease? a. Pain with diaphoresis b. Discomfort in one digit c. Numbness and tingling in the legs d. Cramping in the foot while resting 98. The nurse has completed nutritional counseling with an overweight client about weight reduction to modify the risk for coronary artery disease (CAD). The nurse should determine the teaching is successful if the client states that which weight loss goal is safe? a. One half pound per day b. Two pounds per week c. Four pounds per week d. Six pounds per week 99. The nurse has reinforced instructions to the family of an older client who seems anxious about being discharged after cardiac surgery. The nurse understands further teaching is needed if a family member makes which statement? a. "Recuperation after cardiac surgery is generally slower for older people." b. "It's important to get out of bed every day, even if tired or weak at first." c. "Fatigue, discomfort, and lack of appetite occur more commonly with older people and may last for 2 to 5 weeks." d. "A daily half-mile-long brisk walk generally helps people bounce back more quickly and provides more of a sense of control." 99. The nurse monitors the laboratory data on a client at risk for coronary artery disease. A fasting blood glucose reading of 200 mg/dL is recorded on the chart. The nurse analyzes this result as indicative of which finding? a. Decreased, indicating a decreased risk of coronary artery disease b. Elevated, but would not present a risk for coronary artery disease c. Elevated, signaling the presence of diabetes mellitus, a risk factor of coronary artery disease d. Normal, indicating adequate blood glucose control with no risk for coronary artery disease 101. The nurse has completed counseling about smoking cessation with a client with coronary artery disease (CAD). The nurse determines that the client has understood the material best if the client makes which statement? a. "A smoker has twice the risk of having a heart attack as a nonsmoker." b. "I may try just cutting down first, because the damage has already been done." c. "I don't think I want to quit because none of the effects are reversible anyway." d. "I'm never going to start again because I can cut my risk of cardiovascular disease to zero within a year." 102. The nurse has given simple instructions on preventing some of the complications of bed rest to a client who experienced a myocardial infarction. The nurse should intervene if the client was performing which of these contraindicated activities? a. Deep breathing and coughing b. Repositioning self from side to side c. Isometric exercises of the arms and legs d. Ankle circles, plantar, and dorsiflexion exercises 103. A client with a diagnosis of heart failure (HF) is preparing for discharge to home from the hospital. Which condition indicates the client is ready for discharge to home? a. The client can get the prescriptions filled. b. The client can be self-sufficient at home without any help. c. The client can independently dress and put on support hose. d. The client can verbally describe the daily medications, doses, and times to be administered. 104. A client admitted to the hospital with coronary artery (CAD) disease complains of dyspnea at rest. The nurse determines that which would be of most help to the client? a. Providing a walker to aid in ambulation b. Elevating the head of the bed to at least 45 degrees c. Performing continuous monitoring of oxygen saturation d. Placing an oxygen cannula at the bedside for use if needed Correct 105. The nurse is evaluating the effects of care for the client with deep vein thrombosis. Which limb observations should the nurse note as indicating the least success in meeting the outcome criteria for this problem? a. Pedal edema that is 3+ b. Slight residual calf tenderness c. Skin warm, equal temperature both legs d. Calf girth ⅛ inch larger than unaffected limb 106. A client is at risk for complications of heart failure. Which is the nurse's priority for early detection of the most likely cause of complications with this client? a. Checking vital signs b. Reviewing serum electrolytes c. Evaluating total body fluid d. Monitoring electrocardiogram 107. A female client complains of an "odd, left-sided, twinge-like pain" along the anterior axillary line and states she has had this feeling for the past 3 days. Which is the initial action? a. Administer naproxen (Naprosyn). b. Listen to the client's heart and lungs. c. Determine if the pain is cardiac in origin. d. Ask the client about previous cardiac disease. 108. A client's blood pressure is 100/78 mm Hg; the client has tachycardia and is cool and pale. The nurse assists the client to which position to promote tissue oxygenation and alleviate hypoxia? a. Supine b. Left lateral c. Semi-Fowler's d. Trendelenburg's 109. The nurse notes this rhythm on the client's cardiac monitor. The nurse next reports that the client is experiencing which heart rhythm? Refer to figure. a. Normal sinus b. Atrial fibrillation c. Sinus bradycardia d. Ventricular fibrillation 110. The client's B-type natriuretic peptide (BNP) level is 691 pg/mL. Which intervention should the nurse institute when providing care for the client? a. Take daily weights and monitor trends. b. Encourage fluids to improve hydration. c. Elevate the legs above the level of the heart. d. Position supine with the head of the bed at 30 degrees. 111. A hypertensive client who has been taking metoprolol (Lopressor) has been prescribed to decrease the dose of the medication. The client asks the nurse why this must be done over a period of 1 to 2 weeks. In formulating a response, the nurse incorporates the understanding that abrupt withdrawal could affect the client in which way? a. Result in hypoglycemia b. Give the client insomnia c. Precipitate rebound hypertension d. Cause enhanced side effects of other prescribed medications 112. A client is admitted to the hospital with a venous stasis leg ulcer. The nurse inspects the ulcer expecting to note which observation? a. The ulcer has a pale-colored base. b. The ulcer is deep, with even edges. c. The ulcer has little granulation tissue. d. The ulcer has a brownish or "brawny" appearance. 113. A client has just returned from the cardiac catheterization laboratory. The left femoral vessel was used as the access site. After returning the client to bed, the nurse places a sign above the bed stating that the client should remain on bed rest and in which position? a. In semi-Fowler's position b. With the head of the bed elevated 45 degrees c. With the head of the bed elevated no more than 15 degrees d. With the foot of the bed elevated as much as tolerated by the client 114. A client's serum calcium level is 7.9 mg/dL. The nurse is immediately concerned, knowing that this level could lead to which complication? a. Stroke b. Cardiac arrest c. High blood pressure d. Urinary stone formation 115. A client has a history of left-sided heart failure. The nurse should look for the presence of which finding to determine whether the problem is currently active? a. Presence of ascites b. Bilateral lung crackles c. Jugular vein distention d. Pedal edema bilaterally 116. The nurse is told during shift report that a client is having occasional ventricular dysrhythmias. The nurse reviews the client's laboratory results, recalling that which electrolyte imbalance could be responsible for this development? a. Hypokalemia b. Hypernatremia c. Hypochloremia d. Hypercalcemia 117. A licensed practical nurse (LPN) is assisting in the care of a client who is having central venous pressure (CVP) measurements taken by the registered nurse (RN). The LPN should assist the RN by placing the bed in which position for the reading? a. Flat b. Semi-Fowler's c. Trendelenburg's d. Reverse Trendelenburg's 118. The nurse is assisting a client who will wear a Holter monitor for continuous cardiac monitoring over the next 24 hours. The nurse takes which action to assist the client? a. Shaves the front of the client's chest b. Gives the client a device holder to wear around the waist c. Teaches the client to rest as much as possible during the next 24 hours d. Tells the client to cover the monitor in plastic wrap before taking a bath 119. A client is admitted with an arterial ischemic leg ulcer. The nurse expects to note that this ulcer has which typical characteristic? a. Dark, pink base b. Deep and painful c. Accompanied by very slight pain d. Brown pigmentation of surrounding skin 120. The nurse is assisting in the care of a client with myocardial infarction who should reduce intake of saturated fat and cholesterol. The nurse should help the client comply with diet therapy by selecting which food items from the dietary menu? a. Cheeseburger, pan-fried potatoes, whole kernel corn, sherbet b. Pork chop, baked potato, cauliflower in cheese sauce, ice cream c. Baked haddock, steamed broccoli, herbed rice, sliced strawberries d. Spaghetti and sweet sausage in tomato sauce, vanilla pudding (with 4% milk) 121. The nurse is assisting a client admitted to the hospital with pulmonary edema to prepare for discharge. The nurse should reinforce with the client the importance of complying with which measure to prevent a recurrence? a. Weigh self every morning before breakfast. b. Sleep with the head elevated on only one pillow. c. Adjust diuretic dose based on severity of peripheral edema. d. Take additional digoxin (Lanoxin) if respiratory distress occurs. 122. The nurse is assisting in the care of a client diagnosed with rheumatic heart disease. The nurse should reinforce instructions to the client to notify the dentist before dental procedures for which reason? a. The client requires prophylactic antibiotics before treatment. b. The dentist should use a low-speed drill to avoid dysrhythmias. c. The dentist should use a lidocaine solution without epinephrine. d. The client is at risk for episodes of heart failure triggered by stressful events. 123. A client with a history of angina pectoris complains of substernal chest pain. The nurse checks the client's blood pressure and administers nitroglycerin 0.4 mg sublingually. Five minutes later, the client is still experiencing chest pain. If the blood pressure is still stable, the nurse should take which action next? a. Administer another nitroglycerin tablet. b. Apply 1 to 3 L/minute of oxygen via nasal cannula. c. Call for a 12-lead electrocardiogram (ECG) to be performed. d. Wait an additional 5 minutes, then give a second nitroglycerin tablet. 123. The health care provider is discharging a client with a diagnosis of chronic heart failure. Which health maintenance instructions should the nurse reinforce in the discharge teaching plan? Select all that apply. a. Obtain annual influenza vaccination. b. Restrict fluid intake to 1000 mL per day. c. Avoid adding salt to foods or in cooking. d. Report a weight gain of 3 or more pounds in a week. e. Take an extra dose of prescribed diuretic for swollen ankles. 125. The nurse is preparing for a health fair about tobacco use and the development of coronary heart disease. Which information should the nurse include? Select all that apply. a. R b. Nicotine decreases oxygen to the heart. c. Hypnosis may be helpful to stop smoking. d. Avoid exposure to environmental tobacco smoke. e. Cigars or pipes are healthier than cigarette smoking. f. Tobacco smoking increases a female's level of estrogen. 126. The nurse is caring for a client with a new onset of atrial fibrillation. Which prescribed treatments should the nurse expect? Select all that apply. a. Defibrillation b. Digoxin (Lanoxin) c. Warfarin (Coumadin) d. Electrical cardioversion e. Amiodarone (Cordarone) 127. A client with hyperlipidemia is seen in the clinic for a follow- up visit. Which dietary modifications should the nurse include to lower the risk of coronary heart disease? Select all that apply. a. Use liquid vegetable oil. b. Increase intake of fruits. c. Choose whole grain foods. d. Remove skin from poultry. e. Select whole milk products. 127. The LPN/LVN is caring for a client with left-sided heart failure. Which clinical signs are most important for the nurse to communicate to the health care provider? Select all that apply. a. Pink-tinged frothy sputum b. Increase in respiratory rate c. Ankle and lower leg swelling d. Paroxysmal nocturnal dyspnea e. Auscultation of crackles throughout the lungs 129. The nurse is admitting a client with acute pericarditis who reports chest pain. When planning the client's care, which position should the nurse encourage the client to assume to alleviate the chest pain? Select all that apply. a. Lying supine b. Right side-lying c. Sitting up and leaning forward d. Semi-Fowler's with knees bent e. Head of bed elevated to 45 degrees 130. The health care provider is discharging a client with a diagnosis of primary hypertension. Which health maintenance instructions should the nurse reinforce in the discharge teaching plan? Select all that apply. a. Monitor the blood pressure at home. b. Restrict sodium intake as prescribed. c. Take a calcium supplement to lower blood pressure. d. Eye examinations with an ophthalmoscope should be routine. e. Follow-up appointments for blood pressure checks are important 131. The nurse is planning care for a client with diabetes mellitus who has gangrene of the toes to the midfoot. Which goal should be included in this client's plan of care? a. Restore skin integrity. b. Prevent infection. c. Promote healing. d. Improve nutrition. 132. The LPN/LVN is conducting an osteoporosis screening clinic at a health fair. What information should the nurse provide to individuals who are at risk for osteoporosis? (Select all that apply.) a. Encourage alcohol and smoking cessation. b. Suggest supplementing diet with vitamin E. c. Promote regular weight-bearing exercises. d. Implement a home safety plan to prevent falls. e. Propose a regular sleep pattern of 8 hours nightly. 133. An 81-year-old male client has emphysema. He lives at home with his cat and manages self-care with no difficulty. When making a home visit, the nurse notices that this client's tongue is somewhat cracked and his eyeballs appear sunken into his head. Which nursing intervention is indicated? a. Help the client determine ways to increase his fluid intake. b. Obtain an appointment for the client to have an eye examination. c. Instruct the client to use oxygen at night and increase the humidification. d. Schedule the client for tests to determine his sensitivity to cat hair. 134. The nurse is assessing a client who presents with jaundice. Which assessment finding is most important for the nurse to follow up? a. Urine specific gravity of 1.03 b. Frothy, tea-colored urine c. Clay-colored stools d. Elevated serum amylase and lipase levels 135. Which content about self-care should the LPN/LVN include in the teaching plan of a female client who has genital herpes? (Select all that apply.) a. Encourage annual physical and Pap smear. b. Take antiviral medication as prescribed. c. Use condoms to avoid transmission to others. d. Warm sitz baths may relieve itching. e. Use Nystatin suppositories to control itching. f. Use a douche with weak vinegar solution to decrease itching. 135. The LPN/LVN is interviewing a client who is taking interferon alfa-2a (Roferon-A) and ribavirin (Virazole) combination therapy for hepatitis C. The client reports experiencing overwhelming feelings of depression. Which action should the nurse implement first? a. Recommend mental health counseling. b. Review the medication actions and interactions. c. Assess for the client's daily activity level. d. Provide information regarding a support group. 137. A client in the emergency department is bleeding profusely from a gunshot wound to the abdomen. In what position should the nurse immediately place the client to promote maintenance of the client's blood pressure above a systolic pressure of 90 mm Hg? a. Place the client in a 45-degree Trendelenburg position to promote cerebral blood flow. b. Turn the client prone to place pressure on the abdominal wound to help staunch the bleeding. c. Maintain the client in a supine position to reduce diaphragmatic pressure and visualize the wound. d. Put the client on the right side to apply pressure to the liver and spleen to stop hemorrhaging. 138. The nurse assesses a client who has been prescribed furosemide (Lasix) for cardiac disease. Which electrocardiographic change would be a concern for a client taking a diuretic? a. Tall, spiked T waves b. A prolonged QT interval c. A widening QRS complex d. Presence of a U wave 139. When a nurse assesses a client receiving total parenteral nutrition (TPN), which laboratory value is most important for the nurse to monitor regularly? a. Albumin b. Calcium c. Glucose d. Alkaline phosphatase 140. A 62-year-old woman who lives alone tripped on a rug in her home and fractured her hip. Which predisposing factor most likely contributed to the fracture in the proximal end of her femur? a. Failing eyesight resulting in an unsafe environment b. Renal osteodystrophy resulting from chronic kidney disease (CKD) c. Osteoporosis resulting from declining hormone levels d. Cerebral vessel changes causing transient ischemic attacks HESI PN Practice Exam & Questions (Quizlet Review) https://quizlet.com/68815544/hesi-pn-practice-exam-and-questions-flash- cards/ 1. The nurse is planning care for the a client who has fourth degree midline laceration that occurred during vaginal delivery of an 8 pound 10 ounce infant. What intervention has the highest priority? A. Administer Prescribed stool softner B. Administer prescribed PRN sleep medications. C. Encourage breastfeeding to promote uterine involution D. Encourage use of prescribed analgesic perineal sprays. 2. The nurse is palpating the right upper hypochondriac region of the abdomen of a client. What organ lies underneath this area. A. Duodenum B. Gastric Pylorus C. Liver D. Spleen 3. A client comes to the antepartal clinic and tells the nurse that she is 6 weeks pregnant. Which sign is she most likely to report? A. Decreased sexual libido B. Amenorrhea C. Quickening D. Nocturia 4. A client's daughter phones the charge nurse to report that the night nurse did not provide good care for her mother. What response should the nurse make? A. Ask for a description of what happened during the night B. Tell the daughter to talk to the unit's nurse manager C. Reassure the daughter that the mother will get better care. D. Explain that all the staff are doing the best they can. 5. A hosptitalized toddler who is recovering from a sickle cell crisis holds a toy and say's "mine". According to Erikson's theory of psychosocial development, this child's behavior is a demonstration of which developmental stage? A. Autonomy vs. Shame and doubt. B. Industry vs. Inferiority C. intiative vs. Guilt D. Trust vs. Mistrust 6. Which action should the nurse implement in caring for a client following an electroencephalogram (EEG)? A. Monitor the client's vital signs q4h B. Assess for sensation in the client's lower extremities C. Instruct the client to maintain bed rest for eight hours D. Wash any paste from the client's hair and scalp 7. The nurse is caring for a 75- year-old male client who is beginning to form a decubitus ulcer at the coccyx. Which intervention will be most helpfull in preventing further development of the decubitus? A. Encourage the client to eat foods high in protein B. Assess the client with daily range of motion exercises C. Teach the family how to perform sterile wound care D. Ensure the IV fluids are administered as prescribed 8. What is the homeostatic cellular transport mechanism that moves water from a hypotonic to a hypertonic fluid space? A. Filtration B. Diffusion C. Osmosis D. Active transport 9. The nurse is taking blood presure of a client admitted with a possible myocardial infarction. When taking the client's BP at the brachial artery, the nurse should place the client's arm in which position? A. Slightly above the level of the heart B. At the level of the heart C. At the level of comfort for the client D. Below the level of the heart 10. What are the final parameters that produce blood pressure? (select all that apply) A. Heart rate B. Stroke volume C. Peripheral resistance D. Neuroendocring hormones E. Muscle tone 11. A client begins an antidepressant drug during the second day of hospitalization. Which assessment is most important for the nurse to include in this client's plan of care while the client is taking the antidepressant? A. Appetite B. Mood C. Withdrawl D. Energy level 12. Based on the documentation in the medical record, which action should the nurse implement next? A. Give the rubella vaccine subcutaneously B. Observe the mother breastfeeding her infant C. Call the nursery for the infant's blodd type result D. Administer Vicodin one tablet for pain 13. A client is adminitted to the hosptial with a diagnosis of Pneumonia. Which intervetion should the nurse implement to prevent complications associated with Pneumonia? A. Enourage mobilization and ambulation B. Encourage energy conservation with complete bed rest C. Provide humidified oxygen per nasal cannula D. Restrict PO and intravenous fluids 14. The practical nurse is preparing to administer a prescription for cefazolin (kefzol) 600 mg IM every 6 hours. The available vial is labeled, "Cefazolin (Kefzol) 1 gram and the instrutions for reconsittution, "For IM use add 2ml sterile water for injection. Total volume after reconstruction = 2.5 ml. "when reconstituded, how many milligrams are in each mil of solutions (Enter numeric value only) A. 15 15. Which nursing activity is within the scope of practice for the practical nurse? A. Complete an admission assessment in the normal newborn nursery. B. Discontinue a central venous catheter that has become dislodged C. Observe a client rotate the subcutaneous site for an insulin pump D. Monitor a continuous narcotic epidural for a postoperative client 16. After morning dressing changes are completed, a male client who has paraplegia contaminates his ischial decubiti dressing with a diarrheal stool. What activity is best for the nurse to assign to the unlicensed assistive personnel? A. Identify the need for additional supplies to provide an extra dressing change B. Provide perianal care and collect clean linens for the dressing change C. Document the diarrhea that necessitates an additional dressing change D. Position the client for access to the decubiti sties and remove dressings 17. The nurse is planning to evaluate the effectiveness of several drugs administered by different routes. Arrage the routes of administration in the order from fastest to slowest rate of absorption. A. Intravenous B. Sublingual C. Intramuscular D. Subcutaneous E. Oral 18. A 26-year-old gravida 4, para 0 had a spontaneous abortion at 9 weeks gestation. At one-house post dilation and curettage (D&C) the nurse assess the vital signs and vaginal bleeding. The client begins to cry softly. How should the nurse intervene? A. Offer to call the social worker to discuss the possiblity of abortion B. Reassure the client that the infertility specialist can help C. Express sorrow for the client's grief and offer to sit with her D. Chart the vital signs and amount of vaginal bleeding 19. A terminally ill male client and his family are requesting hospice care after discharge from the hosptial and ask the nurse to explain what kind of care they should expect. The nurse should indicate that hospice philosophy focuses on what aspect of health care? A. Enhance symptom management to improve end of life quality B. facilitates assisted suicide with the client's consent C. Offers ways to postpone the death experience at home D. Provide training for family members to care for the client. 20. The nurse observes a wife shaving her husband's beard with a safety razor by holding the skin taut and shaving in the direction of the hair growth . What action should the nurse take? A. Advsie the wife to shave against the hair growth B. Teach the wife to keep the skin loose to avoid cuts C. Encourage the wife to continue shaving her husband D. Demonstrate the correct procedure to the wife 21. To assess pedal pulse what arterial sites should the nurse palpate? (select all that apply) A. Posterior tibialis artery B. Politeal artery C. External femoral artery D. Dorsalis pedis artery E. E Radial artery 22. The nurse is admitting a client who is diagnosed with Angina Pectoris. Which precipitating factor in this client's history is likely to be related to the anginal pain? A. Smokes one pack of cigarettes daily B. Drinks two beers daily C. Works in a job that requires exposure to the sun D. Eats while lying in bed 23. The nurse is assessing an older resident of a long-term care facility who has a history of Benign Prostatic Hypertrophy and identifies that the client's bladder is distended. The healthcare provider prescribes post-voided residual catherterization over the next 24 hours and placement of an indwelling catheter if the residual volume exceeds 100 mL. The client's PO intake is 600 mL, and fifteen minutes ago, the client voided 90 mL. What action should the nurse take? A. Stand the client to void and run tap water within hearing distance before catheterizing the client. B. Straight catheterize and if the residual uring volume is greater than 100 mL, clamp catheter C. Catheterize q2H and place in an indwelling catheter at the end of the prescribed 24hr period. D. Catheterize with an indwelling catheter and if the residual volume is greater than 100 mL. Inflate the balloon. 24. A client is receiving dexamethasone (Hexadrol, Decadron). What symptoms should the nurse recognize as Cushionoid side effects? A. Moon face, Slow wound healing, muscle wasting sodium and water retention B. Tachycardia hypertension, weight loss, heat intolerance, nervousness, restlessness, tremor C. Bradycardia, weight gain, cold intolerance, myxedema facies and periobarbital edema D. Hyperpigmentation, hyponatremia, hyperkalemia, dehydration, hypotension 25. The cervix is the opening into the uterine cavity. What is its function in reproduction? A. Accepts and interprets signals of sexual stimuli B. Secretes mucus to facilitate sperm transport C. Serves as the site for union of ovum ans sperm D. Receives the penis during intercourse 26. The nurse is working in a community health setting and assisting the charge nurse in performing health screenings. Which individual is at highest risk for contracting an HIV infection? A. 17-year-old who is sexually active simultaneously with numerous partners B. 34-year old homosexual who is in a monogamous relationship C. 30-year-old cocaine user who inhales and smokes drugs D. 45-year-old who has received two blood transfusions in the past 6 months 27. The nurse is administering amiodarone (Cordarone) to a client who has been admitted with Atrial Fibrillation (AFIB). What therapeutic response should the nurse anticipate? A. Conversion of irregular heart rate to regular heart rhythm B. Pulse oximetry readings within normal range during activity C. Peripheral pulse points with adequate capillary refill D. Increase excercise tolerance without shortness of breath 28. An elderly male client is planning to vacation with a group of senior citizens. He is concerned about developing constipation during the airplane flight. He share this concern with the nurse at the retirement home. Which recommendation is best for the nurse to provide? A. Use an over the counter stool softener when needed B. Eat a high protein diet C. Increase the fluid intake in your diet D. Decrease the fat content in your diet 29. The nurse is assessing a client with dark skin who is in Respiratory Distress. Which client response should the nurse evaluate to determine cyanosis in this particular client? A. Abnormal skin color changes in a client with dark skin cannot be determined B. Blanching the soles of the feet in a client with dark skin reveals cyanosis C. The lips and mucus membranes of a client with dark skin are dusky in color D. Cyanosis in a client with dark skin is seen in the sclera 30. When inserting an indwelling urinary catheter (Foley) in a female client, the nurse observes urine flow into the tubing. What action is taken next? A. Document the color and clarity of the urine B. Insert the catheter an additional inch C. Ask the client to breathe deeply and slowly exhale D. Inflate the balloon with 5mL of sterile water 31. A client has a prescription for a Transcutaneous Electrical Nerve Stimulator (TENS) unit for pain management during the postoperative period following a lumber Laminectomy. What information should the nurse reinforce about the action of this adjuvant pain modality? A. Mild electrical stimulus on the skin surface closes the gates of nerve conduction for sever pain B. Pain perception in the cerebral cortex is dulled by the unit's discharge of an electrical stimulus C. An infusion of medication in the spinal canal will block pain perception D. The discharge of electricity will distract the client's focus on the pain 32. Based on the Nursing diagnosis of "Potential for infection related to second and third degree burns," which intervention has the highest priority? A. Application of topical antibacterial cream B. Use of careful hand washing technique C. Administration of plasma expanders D. Limiting visitors to the burned client. 33. The mother of an 8-year-old boy tells the nurse that he fell out of a tree and hurt his arm and shoulder, which assessment finding is the most significant indicator of possible child abuse? A. The child looks at the floore when answering the nurse's questions B. The mother's version of the injury is different from the child's version C. The child has several abrasions on the chest and legs D. The mother refuses to answer questions about family history 34. A client has a prescription for enteric-coated (EC) aspirin 325mg PO daily. The medication drawer contains one 325mg aspirin. What action should the nurse take? A. Contact the pharmacy and request the prescribed form of aspirin B. Instruct the client about the effects when given the medication C. Administer the aspirin with a full glass of water or a small snack D. Withhold the aspirin until consulting with the healthcare provider 35. The nurse explains the 2-week dosage prescription of prednison (Deltasone) to a client who has poison ivy over multiple skin surfaces. What should the nurse emphasize about the dosing schedule? A. Decrease dosage daily as prescribed B. Monitor oral temperature daily C. Take the prednison with meals D. Return for blood glucose monitoring in one week 36. The nurse is preparing to administer a 1.2mL injection to a 4- year-old. Which are the best sites to administer an IM injection? Select all that apply. A. Vastus lateralis B. Ventrogluteal C. Dorsogluteal D. Rectus femoris E. Deltoid 37. Which nonfood item is the most common cause of respiratory arrest in young children? A. Broken rattles B. Buttons C. Pacifiers D. Latex balloons 38. A new mother is at the clinic with her 4-week old for a well baby check up. The nurse should tell the mother to anticipate that the infant will demonstrate which millstone by 2-months of age. A. Turns from side to back and returns B. Consistently returns smiles to mother C. Finds hands and plays with fingers D. Holds head up and supports weight with arms 39. The nurse is monitoring a client's intravenous infusion and observes that the venipuncture site is cool to the touch, swollen and teh infusion rate is slower than the prescribed rate. What is the most likely cause of this finding? A. The solution's rate is too rapid B. The client has phlebitis C. The infusion site is infected D. The infusion site is infiltrated 40. The nurse observes that a male client's urinary catheter (Foley) drainage tubing is secured with tape to his abdomen and then attached to the bed frame. What action should the nurse implement? A. Raise the bed to ensure the drainage bag remains off the floor B. Attach the drainage bag to the side rail instead of the bed frame C. Observe the appearance of the urine in the drainage tubing D. Secure the tubing to the client's gown instead of his abdomen 41. In assisting a client to obtain a sputum specimen, the nurse observes the client cough and spit a large amount of frothy saliva in the specimen collection cup. What action should the nurse implement next? A. Advise the client that suctioning will be used to obtain another specimen B. Re-instruct the client in coughing techniques to obtain another specimen C. Provide the client a glass of water and mouthwash to rinse the mouth D. Label the container and place the container in a bio-hazard transport bag 42. After report, the nurse receives the laboratory values for 4 clients. Which client requires the nurse's immediate intervention? The client who is..... A. short of breath after a shower and has a hemoglobin of 8 grams B. Bleeding from a finger stick and has a prothrombin time of 30 seconds C. Febrile and has a WBC count of 14,000/mm3 D. Trembling and has a glucose level of 50 mg/dL 43. 4 hours after administration of 20U of regular insulin, the client becomes shakey and diaphoretic. What action should the nurse take? A. Encourage the client to excercise B. Administer a PRN dose of 10U of regular insulin C. Give the client crackers and milk D. Record the client's reaction on the diabetic flow sheet 44. The nurse is changing the colostomy bag for a client who is complaining of leakage of diarrheal stool under the disposable ostomy bag. What action should the nurse implement to prevent leakage? A. Place a 4X4 wick in the stoma opening B. Apply a layer of zinc oxide ointment to the perimeter of the stoma C. Cut the bag opening to the measurement of the stoma size D. Administer a PRN antidiarrheal agent 45. Prior to administering morphine sulfate (Morphine), the nurse takes the client's vital signs. Based on which finding should the nurse withhold administration of the medication until the charge nurse is notified? A. Temperature of 100.8F B. A pulse rate of 150 beats per minute C. A respiratory rate of 10 breaths per minute D. A blood pressure of 180/110 46. Following an open reduction of the tibian, the nurse notes fresh bleeding on the client's cast. Which intervention should the nurse implement? A. Assess the client's hemoglobin to determine if the client is in shock B. Call the surgeon and prepare to take the client back to the operating room C. Outline the area with ink and check it q15 minutes to see if the area has increased D. No action is required since postoperative bleeding can be expected 47. The nurse is with a client when the healthcare provider explains that the biopsy classifies the results as a T1N0M0 tumor. Later in the morning, the client asks the nurse, "what do these letters T1N0M0, stand for?" which response should the nurse provide first? A. "The letters are used to predict the prognosis of the cancer or tumor." B. "The letters stand for tumor size, node involvement and metastasis." C. "Let me refer you to the charge nurse." D. "Are you confused? Would you like to talk?" 48. The nurse plans to administer the rubella vaccine to a postpartum client whose titer is < 1:8 and who is breastfeeding? what information should the nurse provide this client? A. The client should bottle feed and pump her breast for 3 days following immunization B. The vaccine is given to produce maternal antibodies before lactation occurs C. The infant will receive immunization through the mother's breast milk D. The client should not get pregnant for 3 months after immunization 49. In counting a client's radial pulse, the nurse notes the pulse is weak and irregular. To record the most accurate heart rate, what should the nurse take? A. Recheck the radial pulse in thirty minutes B. Palpate the radial pulse for thiry seconds and double the rate C. Count the apical pulse rate for sixty seconds D. Compare the radial pulse rate bilaterally and record the higher rate. 50. Which structures are located in the subcutaneous layer of the skin? A. Sebaceous and sweat glands B. Melanin and Keratin C. Sensory receptors and hair follicles D. Adipose cells and blood vessels 51. The nurse in charge of a Nursing unit in a long term care facility. Which task is best for the nurse to assign to an unlicensed assistive personnel (UAP) who is helping with the care of several clients? A. Measure the amount of a client's residual urine after voiding B. Cleanse the perineal area of a client with urinary incontinence C. Insert a straight catheter to obtain a urine specimen for culture D. Provide catheter care for a client with a suprapubic catheter 52. A client requires application of an eye shield to the right eye. What should the nurse do in order to apply tape in which direction to anchor the shield most effectively? A. Across the eye from the bridge of the nose to the right temple B. Longitudinally from the right forehead to the right cheek C. From the mid-forehead over to the right zygomatic process D. From the right lateral forehead surface to the medial nasal crease 53. 36 hours after delivery, the nurse determines a client's fundus is just above the umbilicus and displaced to the right of midline. What action should the nurse take first? A. Palpate the bladder for distention B. Ask the client when her last bowel movement occurred C. Catheterize the client and record the amount D. Assess the amount of lochia 54. A client presents in the clinic because of generalized swelling after a bee sting. What intervention should the nurse implement first? A. Assess site of sting and remove stinger if present B. Perform mini-mental status exam to assess level of consciousness C. Determine respiratory status and apply a pulse oximeter D. Attach electrodes to monitor cardiac rhythm 55. The nurse is administering multiple medications to a 78-year-old client because of problems related to polypharmacy. At this client's age, which assessment is most important for the nurse to make? A. Cumulative serum drug levels and toxicity B. Synergistic actions due to simultaneous administration C. Tolerance to drugs that have been taken for long periods of time D. Antagonist actions of multiple medications 56. In obtaining an orthostatic vital sign measurement, what action should the nurse take first? A. Count the client's radial pulse B. Apply a blood pressure cuff C. Instruct the client to lie supine D. Assist the client to stand upright 57. A 3-week-old infant is admitted for surgical repair of Pyloric Stenosis. What interventions should the nurse expect to implement to establish hydration in the immediate postoperative period? A. Diaper weights and urin specific gravity B. Gastronomy feedings in supine position C. Nipple feedings with glucose water D. Gavage feedings with 15mL of formula 58. Urinary catheter (Foley) with a 5mL inflated balloon is being removed by the nurse. After withdrawing 5 mL of fluid from the balloon, the nurse begins to withdraw the catheter while the client is in a Semi-Fowler's position. However, the nurse meets resistance and the clients voices discomfort. What action should the nurse take next? A. Attempt to withdraw additional fluid from the balloon B. Assist the client in taking a series of deep breaths C. Lower the head of the client's bed so the client is supine D. Allow the client to rest before continuing to remove the catheter 59. The home health nurse observes an elderly male client attempt to open a child-proof medication container. When he is unsuccessful in opening the container, he throws it across the room and curses loudly. What action should the nurse implement? A. Transfer the medications to another bottle that is easier to open B. Leave the client's home immediately and plan to return later C. Igonore the outburst and demonstrate how to open the bottle D. Describe other types of medication containers that are available 60. At 7AM, a Diabetic client is conscious with a serum glucose level of 50mg/dL. To manage this client's care effectively, what should the nurse administer? A. Orange juice B. Glucagon C. 10 units of regular insulin f. IV of 5% glucose in water at 100 mL/hr 61. A nurse is caring for a client with Multiple Sclerosis (MS) who is receiving an immunosuppressant. Which action is most important for the nurse to implement to evaluate for adverse effects from this particular medication? A. Observe the client's skin for bruising B. Auscultate the client's bowel sounds C. Monitor the clients intake and output D. Note changes in the client's weight 62. A male client with Hypercholesterolemia is being discharged with a new prescription for simvastatin (Zocor). The client tells the nurse that he understands it is important to have liver tests performed periodically. How should the nurse respond? A. Instruct the client that the only regular testing needed is to monitor his cholesterol level B. Teach the client that liver test are usually only done if the client reports symptoms C. Review with the client that renal function tests are needed, rather than liver tests D. Confirm that the client correctly understands the need to monitor liver function regularly 63. An obese female client with a high serum cholesterol level comes to the clinic for a follow-up evaluation. She tells the nurse that she is now walking 30 minutes three times per week and is eating a carbohydrate free, high protein diet in order to lose weight. What response is best for the nurse to provide? A. Explain to the client that her diet choice is not helpful in lowering cholesterol levels B. Discuss the importance of maintaining a target heart rate during each exercise period C. Teach the client additional ways to lower cholesterol, including stress management D. Praise the client for her exercise and dieting efforts and encourage her to continue with this program 64. A child with Chronic Asthma is scheduled for Chest Physiotherapy. When should the nurse administer the meter-dosed inhalar (MDI) puff of bronchodilator relative to postural drainage treatments? A. Before postural drainage B. During postural drainage C. After postural drainage D. Between treatments 65. A client has a prescription for lorazepam (ativan) 1 mg for anxiety. The medication is supplied as 0.5mg tablets. How many tablets should the client take? (enter numeric value only. 2 66. The nurse is caring for a middle-aged client who had a Myocardial infarction (MI) 3 days ago. Which finding is most important for the nurse to report? A. Frothy red-tinged sputum B. Irregular heart rate C. Two pound weight gain D. Dependent edema 67. A client is diagnosed with Clostridium Difficile (CDIFF). What action should the nurse implement to prevent the spread of the organism? A. Place a surgical mask on the client during transport B. Don non-sterile gloves when performing direct care C. Wear a particular respirator mask when in the room D. Keep the door closed to the client's room at all times 68. A 67-year-old woman who lives alone tripped on a rug in her home and fractured her right hip. The nurse knows that which predisposing factor contributes to the occurrence of hip fractures among elderly women. A. Urinary retention resulting in renal calculi formation B. Failing eyesight resulting in an unsafe environment C. Osteoporosis resulting from hormonal changes D. Transient ischemic attacks (TIAs) which impair mental activity 69. An elderly client is admitted for evaluation of Alzheimer's disease. At 2AM, the nurse finds the client trying to open the emergency door. What is the most appropriate response for the nurse to make in this situation? A. "This is the emergency door. Are you looking for the bathroom?" B. "You look confused. Would you like to talk about your feelings?" C. "Let's go back to your room. Your doctor does not want you to be walking alone." D. "You want to go outside at this time of night? It's dangerous out there." 70. Which nurse's behavior is a breach of client confidentiality according to the Health Insurance Portable Accountability Act (HIPPA) regulations? A. A daily report sheet with the information of the team's clients is taken home. B. Privileged health information (PH) is mailed through the US postal service C. A client is called by both the first and last name in a public waiting room. D. The ambulance health care provider is given information about the client's history 71. A client is returning to the surgical unit after a total right knee replacement. Which assessment findings are most important for the nurse to include in this client's record? A. Pedal pulses, pallor, pain, paresthesia or paralysis B. Level of consciousness, lung sounds, and bladder tone C. Swallow reflex, nausea, and vomiting and IV infusion rate D. Call bell side rails, bed in position, and ambulation aids 72. The nurse is standing at the clinic desk when a mother and preschool child approach. The mother tells the nurse that her child has a fever and rash. What action should the nurse take? A. Take the child immediately to a different part of the clinic B. Have them wait in the waiting area away from the other children C. Tell the mother to return to the clinic when the rash subsides D. Place them first on the list to see the healthcare practitioner 73. A nurse is contributing to a care plan for an adolescent female client with Anorexia Nervosa. Which outcome statement or goal would be most appropriate for this client? A. A She will participate in a daily aerobic exercise program B. She will consume at least 50 percent of all meals C. Her laboratory values will remain within normal limits D. She will develop a positive body image and self-identity 74. A female client with no family history of Breast Cancer (BA) asks the nurse how often she should obtain a Mammogram. Which additional client information should the nurse obtain before answering this client's question? A. Current age B. Breast size C. Breastfeeding history D. Menopausal status 75. The nurse is working on the postpartum unit and is assisting a new mother with her newborn's diaper change. The mother states that the infant fed well and completed the whole bottle of formula. What action should the nurse implement first when the infant begins to spit up during the diaper change? A. Bubble or burp the infant by patting the infant's back B. Encourage the mother to avoid over feeding the infant C. Turn the newborn and bulb suction the mouth and nose D. Wipe away the secretions and finish the diaper change 76. An older male client tells the nurse that his religion does not permit him to bathe daily. How should the nurse respond? A. Review the importance of hygienic measures for improved health B. State that the healthcare provider has prescribed a bath today C. Offer the client several choices of times to bathe during the day D. Request that the client clarify his religious beliefs about bathing 77. A new father asks the nurse the reason for placing an ophthalmic ointment in his newborn's eyes. What information should the PN provide? A. Possible exposure to an environmental staphylococcus infection can infect the newborn's eyes and cause visual deficits B. The newborn is at risk for blindness from a corneal syphilitic infection acquired from a mother's infected vagina C. Treatment prevents tear duct obstruction with harmful exudate from a vaginal birth that can lead to dry eyes in the newborn D. State law mandates all newborns receive prophylactic treatment to prevent gonorrheal or chlamydial ophthalmic infection 78. The scope of practice for the practical nurse includes which client assessments? A. An agitated client with bilateral wrist restraints B. New admission of a client with deep vein thrombosis C. Return of a postaneshesia client following a colon resection D. Transfer of a client with sepsis from a long-term care facility 79. What skin care measure should the nurse implement for a client who underwent an external radiation treatment the previous day? A. Cleanse the radiated area with water and pat the skin dry B. Lightly massage the radiated skin with a lanolin-based lotion C. Rinse the site with normal saline and cover with a sterile towel D. Use of soft washcloth to gently remove the skin markings 80. Which organ lays retroperitoeally? A. Kidneys B. Testicles C. Urinary bladder D. Pancreas 81. The nurse is caring for a client with Myasthenia Gravis. What time of day is best for the nurse to schedule physical excercises with the physical therapy department? A. Before bedtime, at 2000 B. After breakfast C. Before the evening meal D. After lunch 82. The nurse is planning to ambulate client who has been on bed rest for 24 hours following a Colon Resection. To ambulate this client safely, which intervention should the nurse implement first? A. Place non-skid shoes on the client B. Show the client how to use the call light C. Use a gait belt to support the client D. Assist the client to a bedside sitting position 83. A Client is admitted to the hospital with second and third degree burns to the face and neck. How should the nurse best position the client to maximize function of the neck and face and prevent contracture? A. The neck extended backward using a rolled towel behind the neck B. Prone position using pillows to support both arms outward from the torso C. Side-lying position using pillows to support the abdomen and back D. The neck forward using pillows under the head and sandbags on both sides 84. A client receives a new prescription for the angiotensin II receptor antagonist losartan (Cozaar). Which client instruction should the nurse encourage this client to follow? A. Move slowly when getting up to prevent sudden dizziness B. Take this medication with or after meals C. Do not stop this medication until all of the tablets are gone D. Keep the dietary log during initial therapy 85. The healthcare provider prescribes erythromycin (ilosone) 300 mg PO QID. The medication label reads, "ilosone 100mg/5mL" How many mL should the nurse administer at each does? (Enter the numeric value only) 15 86. The nurse is monitoring a client with an IV infusion in the left antecubital fossae. The infusion pump is functioning without alarms at the prescribed rate of 100mL/hour. The site is warm, red and without swelling. What conclusion should these findings indicate to the nurse? A. The IV fluids are infusing into the subcutaneous tissues and the pump should be stopped B. The infusion pump is functioning properly and the IV site is healthy C. The insertion date should be verified and the IV discontinued D. The site is inflamed and should be reported to the RN for placement in another site. 87. The nurse reviewes the laboratory results of a client whose serum pH is 7.38 on the pH scale what does this value imply about the clients homeostasis A. Alkalosis B. Acidosis C. Normal serum PH D. Incompatible with life 88. The nurse plans to assess a newborn and to check the infant's Moro reflex. In assessing this reflex, the nurse is evaluating which parameter? A. Neurological integrity B. Renal functioning C. Thermogenic regulation D. Respiratory adequacy 89. The nurse assigns an unliscensed assistive personnel (UAP) to feed a client who is at risk for aspirations. To ensure that the task is safely delegated what action should the nurse implement? A. Inform the UAP that the suction is available at the bedside B. Instruct the UAP to notify the PN if the client begins to choke C. Observe the UAP's ability to implement precautions during feed D. Ask the UAP about previous experience performing this skill 90. The unlicensed assistive personnel (UAP) reports to the nurse that a client refused to bathe for the third consecutive day. What action is best for the nurse to take? A. Ask the client why the bath was refused B. Ask family members to encourage the client to bathe C. Explain the importance of good hygiene to the client D. Reschedule the bath for the following day 91. An adult female client is admitted to the psychiatric unit with diagnosis of major depression. After 2 weeks of antidepressant medication therapy, the nurse notices the client has more energy, is giving her belongings away to her visitors, and is in an overall better mood. Which intervention is best for the nurse to implement? A. Tell the client to keep her belongings because she will need them at discharge B. Ask the client if she has had any recent thoughts of harming herself C. Reassure the client that the antidepressant drugs are apparently effective D. Support the client by telling her what wonderful progress she is making. 92. In assisting a client perform pursed lip breathing, the nurse should ensure that the client performs which action? A. Inhale through the nose with the mouth closed and exhale through pursed lips B. Inhale through pursed lips then exhale with the mouth held open C. Inhale through pursed lips and then exhale through the nose with the mouth closed D. Inhale through the mouth puff the cheeks and exhale through pursed lips 93. A 3 year-old admitted with fever of unknown origin (FUO) has begun vomiting in the past half hour. The child's temperature is 101.80 F, and the last dose of antipyretic medication was given 5 hours ago. The child has prescriptions of acetaminophen (Tylenol) 160 MG per 5 mL elixir or 160 mg suppositories PRN fever or pain. What action should the nurse take at this time? A. Make the child NPO and hold all medications untill the vomiting has stopped B. Give acetaminophen elixir to ensure the child's cooperation with swallowing C. Notify the healthcare provider that the child's fever has become dangerously high D. Use an acetaminophen suppository for the fever since the child is vomiting 94. A client is having Radical Masectomy. What is the position of choice during the immediate postoperative period? A. Side-lying on the operative side with the bed flat B. Supine with the arm on the operative side in a dependent position C. Semi-Fowler's position with the arm on the operative side elevated D. Sim's position with the arm on the operative side in a dependent position 95. The nurse assesses the perineum of a client 12 hours after a normal vaginal delivery and finds that she has Perineal Hematomas. The nurse should prepare for which treatment? A. Heat lamp three times per day B. Insertion of vaginal packing C. Cold packs to the perineum D. Operative excision of the hematomas 96. A client at 28 weeks gestation is admitted to the antepartum unit and is being treated for preterm labor. She has a prescription for brethine (Terbutaline) 250 micrograms subcutaneously q4h. The medication is available for injection in 1 mg per ML vials. How many mL should the nurse administer? A. 0.025 B. 0.0025 C. 0.25 D. 25.0 97. A school-aged child with AIDS is exposed to an active case of Varicella. The nurse should recommend that the family take which action? A. Obtain penicillin G 1000U weekly B. Obtain the varicella vaccine C. Enroll in a home school program D. Obtain the varicella zoster immune globulin 98. The principle of client advocacy is best demonstrated when the nurse exhibits which behaviors on behalf of the client? A. Nurse who contacts child protective services to report a mother's decision to refuse vaccination for her firstborn infant B. Nurse refusing to care for a convicted rapist stating that personal discomfort would inhibit provision of quality of care C. Nurse who translates complaints for a Spanish-speaking client to the healthcare provider during rounds D. Nurse sharing information about life after death with a grieving family who just lost a loved one 99. The nurse is preparing a client for an Intravenous Pyelogram (IVP) scheduled for the following morning. What action is most important for the nurse to implement? A. Determine if the client has any allergies to shellfish B. Inform client that an IV dye will be administered before the IVP C. Explain that dizziness may occur when the dye is given D. Administer a bowel prep the evening before the procedure 100. A nurse refuses to perform a procedure because it is beyond the scope of practice for practical nurses. Which resource best defines the nurse's legal responsibility in regard to scope of practice? A. Nursing practice standards for Licensed Practical/Vocational Nurses B. State Nurse Practice Act C. Code of Ethics for Licensed Practical/Vocational Nurses D. Patients Bill of Rights 101. While making the bed of a female client who is sitting in the bedside chair, the nurse observes the client seem anxious. To encourage verbalization by the client, what action should the nurse take? A. Continue to make the bed while conversing with the client B. Sit next to the client at a slight angle to continue the conversation C. Remain standing close enough to the client to hold her hand D. Bring a chair face-to-face with the client for further discussion 102. A client is admitted for observation after experiencing a Transient Ischemic Attack (TIA). The nurse anticipates implementing care for which client problem? A. High risk for injury B. Altered breathing patterns C. Ineffective airway clearance D. High risk infection 103. An elderly postoperative client has the Nursing diagnosis, "Impaired mobility related to fear of falling." Which desired outcome best directs Nursing actions for this client? A. The physical therapist will instruct the client in the use of a walker B. The nurse will place a gait belt on the client prior to ambulation C. The client will ambulate with assistance q4h D. The client will use self-affirmation statements to decrease fear 104. A female client complains to the nurse about being admitted to a semi-private room and expresses her displeasure because she requested a private room prior to admission. What response is best for the nurse to provide this client? A. Room assignments are based on client's acuity level, not necessarily by request B. I will place your name on the room request list for the next available private room C. Your healthcare provider must provide a written request to get you a private room D. There are no private rooms available, so you will have to stay here for the time being. 105. During preoperative preparation, the nurse should offer the client which explanation about why deep breathing exercising with an incentive spirometer are necessary after surgery? A. "Deep breathing exercises using spirometer will help prevent postoperative complications." B. "failure to keep your lungs working may result in pneumonia and death." C. "Incentive spirometry is uncomfortable but necessary for your postoperative care." D. "You will use the spirometer for the first postoperative day only." 106. The nurse is caring for a client who had a total Laryngectomy, Left Radical Neck Dissection, and tracheostomy. The client is receiving Nasogastric (NG) tube feedings via an enteral pump. Today the rate of the feeding was increased from 50mL/hr to 75mL/hr. What parameter should the nurse evaluate the client's tolerance to the rate of feeding? A. Bowel sounds B. Urinary and stool outputs C. Gastric residual volumes D. Daily weight 107. A client is admitted with a fever of undermined origin (FUO). During rounds, the nurse finds the client diaphoretic, and the linens are damp. What should the nurse do first? A. Change the bed linen to prevent chilling B. Check the client's vital signs and pain scale C. Assess the client for urinary incontinence D. Determine fluid intake for the past 8 hours 108. Which client should the nurse assign to an unlicensed assistive personnel (UAP)? A. An older male client with melena who is complaining of abdominal pain and needs a guaic test of a stool sample B. A young adult experiencing flank pain and hematuria who needs all urine strained for stones C. A client who has regular heart rate and after a pacemaker replacement now needs to ambulate D. An elderly client with Right-Sided Hemiplegia and Receptive Aphasia who needs to be transfered to the wheelchair 109. The nurse is administering the shingles vaccine to an older male- client who asks why he should receive the immunization. Which information should the nurse provide? A. A history of chickenpox indicates that the harbors the dormant virus B. The client's last dose of adult immunizations was 10 years ago C. A recent outbreak of fever blisters indicates reactivation of the virus D. Multiple stressful personal experiences increase his risk of shingles 110. In preparing a client for a lumbar puncture, what action should the nurse implement? A. Assist the client to the bathroom to void B. Apply a pulse oximeter to the client's finger C. Teach the client to cough and deep breathing exercises D. Ensure that the client has been NPO for six hours. 111. A client who had a lobectomy two days ago has 2 chest tubes, each attached to a water-sealed drainage system, Pleur-Evac. The nurse observes that in the last 8 hours the serosanguineous fluid has diminished to output in the drainage chamber. What is the most likely outcome of this observation? A. Removal of the lower chest tube, if a chest x-ray reveals no pleural accumulations B. Change the Pleur-Evac system and re-assess output in the empty chamber C. An increase in the prescribed suction force to facilitate-drainage of serosanguineous fluids D. Advance the chest tube to ensure proper placement of the tip to enhance drainage 112. While caring for a client who has been vomiting, the nurse notes that the client's breath has developed a fruity odor. What assessment should the nurse perform first? A. Auscultate the client's bowel sounds B. Determine the client's capillary glucose C. Observe the color of the client's urine D. Measure the client's oxygen saturation 113. The nurse is preparing to assist an elderly client to the bathroom. The nurse knows that an elderly adult's center of gravity changes from the hips to another area of the body. Which area of the body is the center of gravity for the elderly client? A. Upper torso B. Head C. Feet D. Upper extremities 114. A 60 year-old client with cancer of the liver is in Hepatic Coma and unresponsive. What should the nurse say to family members who are inquiring about the condition of their loved one? A. "Your loved one's condition is very critical, and there has been no response in the last 24 hours" B. "The nurses have not been able to arouse the client and the healthcare provider knows the outcome." C. "You need to discuss the condition with the charge nurse in a family conference." D. "The client's condition is extremely critical. Has your family made funeral arrangements?" 115. A client complains of kidney pain. The nurse understands that the kidneys are located where? A. On the retroperitoneal posterior abdominal wall at the costovertebral angle B. Within the curve of the duodenum, posterior to the spleen C. Lateral to the stomach in the hypochondriac region D. Superior aspect of the bladder in right and left iliac region 116. The nurse receives report on an adult client who has a central intravenous (IV) infusion. Where should the nurse observe when assessing the integrity of the access site? A. Umbilical area of the abdomen B. Antecubital fossae of the arm C. Chest wall below the clavicle D. Dorsal surface of the hand 117. The healthcare provider prescribes an IV solution of clindamycin (Cleocin) 850mg in 75 mL of D2W to infuse over 30 minutes. The drop factor is 15 gtt/mL. The nurse should regulate the IV to deliver how many gtt/minute? (Enter numeric value only. if rounding is required round to the nearest whole number) 75mL X 15gtt/mL = 38 38 118. The nurse is administering a subcutaneous injection of epoetin (Epogen) to a client with Chronic Kidney Disease (CKD). This medication is being administered to treat which manifestation of CKD? A. Anemia B. Anuria C. Hypotension D. Edema 119. The nurse is assigned to administer medications in a long-term care facility. A disoriented resident has no identification band or picture. Prior to administering medications to this resident, what is the best Nursing action? A. Confirm the room and bed numbers with those on the medication record B. Ask a regular staf f member to confirm the residents identity C. Hold the medication untill a family member arrives D. Re-orient the resident to name, place and situation. 120. The nurse is assessing an older male client with Gastritis. He has been unable to eat for the past 48 hours and has been vomiting during this same period of time. Which finding can the nurse expect this client to exhibit? A. Edemetous lower extremities and an increased temperature B. A decreased temperature and increased blood pressure C. Dry skin and an increased heart rate D. Diaphoresis and hypertension 121. An adult male client tells the nurse that he believes someone is trying to obtain his computer records, which his wife reports are recreational in nature. The client insists that an elaborate alarm system needs to be installed in his home. The nurse knows that this client is exhibiting which signs or symptom? A. Delusions of persecution B. Ideas of reference C. Hallucinations D. Confabulation 122. The nurse enters a client's room to perform a sterile dressing change. The nurse observes that the client is "gurgling" on oral secretions and coughing. Which action should the nurse take first? A. Position the client supine B. Finger sweep the oral cavity C. Perform oral suctioning D. Provide mouth care 123. What length of blood pressure cuff should be the nurse use when obtaining a client's blood pressure? A. A cuff that is no longer than the circumference of the extremity should be used B. The length of the blood pressure cuff does not make a difference C. The cuf f and its bladder should be nearly encircled in the extremity's circumference D. At least two-thirds the circumference of the extremity should be covered 124. A nurse is assisting a client from the bathroom back to bed following a minor surgical procedure. The client, still not fully alert, reports feeling nauseated and begins to vomit. What is the first action the nurse should take? A. Place a cool rag on the client's head B. Suction the client's oral cavity C. Provide the client an emesis basin D. Place the client in a side-laying position 125. The nurse is caring for a 10-year-old child with hemophilia who has recently been diagnosed as HIV positive. What precautions should the nurst take when interacting with the child and mother? A. No special precautions are needed B. Wear gloves only C. Wear gloves and a mask D. Wear a mask, gloves and gown. 126. A 26 year-old primigravida who delivered a 7-pound male infant 26 hours ago tells the nurse that she is confused about when she and her husband can return to having sexual intercourse. What info should the nurse reinforce with this client? A. They can have intercourse when the episiotomy is healed and the lochial flow has stopped B. They should wait to resume sexual activities until the fatigue assorted with a new baby has passed C. They can resume sexual activity at 6 weeks postpartum D. It is best to wait until both parties feel up to having sexual intercourse 127. The healthcare provider tells the family of a 6-year old child with a malignant brain tumor that the tumor is metastasizing and the child's condition is terminal. How can the nurse best help the family cope with this news? A. Refer the family to a support group to find answers to their questions B. Reinforce the stages of the grieving process C. Listen to the family's reactions and reflect and their fears and concerns D. Transfer the child to a private room 128. The nurse is implementing the plan of care for a client who admits having suicidal thoughts. Which client behavior indicates the highest risk for the client acting on these suicidal thoughts? A. Describes being very depressed B. Has little appetite and neglects personal hygiene C. Is not interested in the activities of family and friends D. Begins to show signs of improvement 129. On a short-staffed unit a long-term care facility, it is important that the nurse assign the unlicensed assistive personnel (UAP) to complete morning care for the resident with which problem first? A. Dyspnea who uses oxygen continously B. Straight catheterization to be performed q6h C. Frequent episidoes of fecal incontinence D. Bolus feeding via PEG tube to be performed q4h 130. The nurse assess a client receiving a hypertonic full strength tube feeding that is infusing continous at 50 mL/hr. Which finding is most important for the nurse to reprot to the charge nurse? A. Dry mucous membranes B. Gastric residual of 50 mL C. Report of increased hunger D. Hyperactive bowel sounds 131. A male client who was admitted with Gangrene of the right lower extremity (RLE) is confused and his wife refuses to sign the operative permit for an above the knee amputation. What action should the nurse take next? A. Explain the consequences of Sepsis if the amputation is delayed B. Notify the RN that the client's wife needs further explanation about the procedure C. Document on the client's record the refusal for surgical treatment D. Enourage the client's wife to express concerns about making the decision 132. A male client attends a community support program for mentally impaired and chemically abusive clients. The client tells the nurse that his drug of choice are cocaine and heroin. What is the greatest health risk for this client? A. Hepatitis B. Hypertention C. Diabetes D. Glaucoma 133. A male client who was admitted with Gangrene of the right lower extremity (RLE) is confused and his wife refuses to sign the operative permit for an above the knee amputation. What action should the nurse take next? A. Explain the consequences of Sepsis if the amputation is delayed B. Notify the RN that the client's wife needs further explanation about the procedure C. Document on the client record the refusal for surgical treatement D. Encourage the client's wife to express concerns about making the decision 134. The nurse is caring for a group of clients on a postpartum unit. After shift report, which client should the nurse assess first? A. Gravida 6 Para 5 who delivered vaginally 24 hours ago B. Gravida 1 Para 0 who is not having contractions C. Gravida 3 Para 3 who delivered vaginally 2 hours ago D. Gravide 1 Para 2 who is preparing for discharge 135. A client returns to the unit following a cardiac catheterization with a Femoral artery Access. Which objective criteria is most important for the nurse to obtain immediately upon the clients return? A. Pupil responses to light B. Pedal pulses C. Respiratory rate D. Peripheral mobility 136. An elderly female client tells the nurse that she does not do regular Breast Self Examinations (BSE) because she is too old. The nurse's response to the client is based on what information? A. The incidence of breast cancer increases with age B. The client should have a health care provider do a breast exam at least once a year C. After age 70, breast cancer is less likely to occur D. The history of breast cancer in a family member is indicative of the need for BSE 137. A client with Meningitis is in a coma and Nursing care includes seizure precautions. To help prevent seizure activity, what interventions should the nurse implement? A. Maintain an oral airway suction equpment and oxygen at the bedside B. Provide respiratory isolation precautions for visitors and staff C. Provide emergency anti convulsant medication at the bedside D. Maintain a quiet calm darkened enviornment 138. The nurse is assisting a female client to obtain a voided specimen for urine culture. After the client cleanses the meatus, which intervention is performed next? A. Initiate the urine stream B. Seperate the labia C. Position the collection cup D. Observe the urine 139. A new protocol for fall prevention is being implemented on the medical unit. During safety rounds, the nurse identifies that an unlicensed assistive personel (UAP) has omitted a vital component of the protocol. After implementing the missing component, what should action should the nurse take? A. Report the UAP's omission to the charge nurse B. Complete an unusual occurence report C. Supervise the UAP after reviewing the protocol D. Assign the UAP to more stable clients the next day 140. What is the best intervention for the nurse to implement when providing morning care for an ambulatory client with an indwelling catheter (Foley)? A. Keep the catheter intact while assisting the client with a shower B. Remove the catheter while the client takes a shower C. Provide the client with a sponge bath in a chair or the bed D. Assist the client with a tub with the catheter clamped 141. Based on the Nursing diagnosis of, "Risk for Infection," which intervention should the nurse implement when providing care for an elderly client with Urinary incontinence? A. Maintain standard precautions B. Utilize an antibacterial perineal wash C. Insert an indwelling urinary catheter D. Initiate contact isolation precautions 142. The charge nurse brings a #18fr urinary catheter (Foley) with a 30 mL balloon to the nurse who is preparing to insert a catheter in a female client who weighs 50 kg. What action should the nurse take first? A. Ask the client if she has previously been catheterized B. Position the client and observe the urinary meatus C. Obtain a 30 ml syringe and a vial of sterile water D. Consult with the charge nurse about the catheter 143. An 82-year old client is admitted to the hospital with a fractured right hip. Following surgical repair, a footboard is placed at the client's feet. What is the reason the nurse will offer concerning the footboard? The footboard is used to... A. Prevent foot drop B. Prevent hip dislocation C. Promote moving in bed D. Promote early ambulation 144. Following a left leg above the knee amputation (AKA), a client voices several complaints. Which statement should be reported to the charge nurse immediately? A. My left foot is so painful B. My incision is so dry C. I've been feeling so light headed D. I'm tired of turning so much 145. In caring for a client following a below the knee amputation (BKA) which task is best for the nurse to delegate to the unlicensed assistive personnel (UAP) who is assisting with the care of this client? A. Empty and measure the drainage in the suction drainage device B. Reassure the client that phantom limb pain is genuine pain C. Review the client's vital signs for indications of infection D. Observe and mark the amount of drainage on the dressing 146. 2 days after an abdominal hysterectomy, an elderly client with diabetes Mellitus Type II has a syncopal episode. Her vital signs are within normal limites and her sugar is 325 mg/dL. what intervention should the nurse implement first? A. Give the client 4 ounces of orange juice B. Administer next scheduled dose of metformin (Glucophage) C. Cancel the clients dinner tray D. Administer regular insulin per sliding scale 147. A client returns to the postoperative unit following an open reduction and internal fixation of a hip fracture. The practical nurse applies the prescribed sequential compression devise (SCD) to both lower extremities. (BLE). What action is important when turning the client to a lateral position? A. Decrease the amount of pressure exerted on both legs while turning the client B. Replace the SCD's with an antiembolic stockings while using an abduction pillow C. Remove both of the SCDs while the cient is turned to the lateral position D. Observe the SCDs continue to inflate and deflate when the client is turned 148. When the nurse asks a male client with Bipolar Disorder if he is going to group session, he responds, "there is no use in me going to that group because all they talk about is Schizophrenia, which doesn't apply to me." Which response is best for the nurse to provide this client? A. "Tell me what medications you are taking right now" B. "You are probably right. The group really does not apply to your condition." C. "It sounds to me like it may be better for you that you stay here" D. "Let's talk about what you may have in common with the other group members." 149. A client is admitted with a newly diagnosed case of active tuberculosis (TB). Which intervention should the nurse teach the client about controlling transmission of tuberculosis (TB)? A. Proper disposal of tissues when coughing B. Importance of an adequate diet C. Complication sof the disease D. Side effects of anti-tubercular medications 150. During CPR, when attempting to ventilate a client's lungs, the nurse notes that the chest is not rising. What action should the nurse take first? A. Reposition the head to ensure an open airway B. Inflate the lungs with more breaths and air pressure C. Finger sweet for a foreign body lodged in the oral cavity D. Reposition hands on chest continue compressions 151. After a change of shift report, the nurse makes rounds on a postoperative unit. Which client finding necessitates the immediate attention of the nurse? A. A client who is having bright red drainage from the rectum following a colonoscopy with a polyp removal B. A client who has pink urine draining from the indwelling urinary catheter following a transurethral prostatectomy C. An older client whose blood pressure is 100/70 after receiving meperidine for pain related to a hip fracture D. A client who has brown green bile draining froma T-tube after a cholecystectomy for Cholelithiasis. 152. Augmentin (amoxicillin/clavulante) 500mg suspension is prescribed for an older adult client who has trouble swallowing . The suspension is available in 125mg/5mL solution. How many ml should the client receive? (enter the numberic value only) 500mg/125m X 5mL = 20mL 153. The nurse observes that there are secretions in the air vent lumen of client's double lumen Nasogastric tube (NGT). Which action should the nurse implement? A. Instill 20 mL of air into the second lumen B. Irrigate the primary lumen with 20 mL of saline C. Place the client in a high Fowler's position D. Turn the suction device to continous suction 154. Which pediatric client is most likely to experience a disturbed body image? A. A. 10-year-old with plantar warts B. B. 14-year-old with acne vulgaris C. C. 16-year-old with a perineal tinea infection D. D. 12-year-old with bacterial cellulitis 155. The first day after a cesarean section (C-Section), when being assisted to the bathroom for the first time, a primavera client experiences a sudden gush of vaginal blood and notices that several blood clots are in the toilet. What action should the nurse take? A. A. Insert an indwelling catheter to empty the bladder and contract the fundus B. B. Return the client to bed and maitain bed rest until the lochial flow slows C. C. Check fundal consistency and continue to monitor the lochial flow amount D. D. Massage the fundus and avoid direct pressure on the cesarean incision. 156. The nurse is emptying the bedpan of a client with a bleeding gastric ulcer. What type of stool can the nurse expect this client to have. A. A. Black tarry stool B. B. Coffee-ground stool C. C. Bright red bloody stool D. D. Clay-colored stool 157. Which structure of the tracheobronchial tree is the most likely to compromise air passage when the smooth muscle layer is affected? A. A. Secondary bronchi B. B. Bronchioles C. C. Segmental bronchi D. D. Alveolar ducsts 158. The nurse is administering routine medications to an assigned group of elderly clients at an extended care facility. Which physiological change commonly associated with aging, increases the elderly client's risk of having an adverse response to the medication? A. A. Decreased gastrointestinal motility B. B. Poor cognitive function C. C. Poor peripheral circulation D. D. Decreased mobility 159. A client with diabetes is admitted with a 1cm size ulcer on the left great toe. The nurse observes that the left foot has a dusky color. In planning the client's care, which intervention should the nurse implement first? A. A. Bathe the wound daily with soap and water B. B. Record the color and temperature of the leg C. C. Perform dorsal flexion and extension exercises D. D. Check the client's dorsalis pedis and posterior tibialis pulse point 160. An ambulatory client with an indwelling urinary catheter (Foley) is requesting to take a shower for the first time. What is the best intervention for the nurse to implement? A. A. Clamp the catheter and assist the client with a tub bath B. B. Keep the catheter intact and assist the client with a shower C. C. Encourage the client to do self-care and provide personal care products D. D. Assist the client with a sponge bath in a chair or the bed 161. The nurse overhears a conversation between an unlicensed assistive personnel (UAP) and another staff member in the hospital cafeteria line concerning a client's reaction to being given a diagnosis of terminal cancer. What is the best Nursing action? A. A. Approach the individuals involved and ask them to stop B. B. Write an incident report and submit it to the unit manager C. C. Tell the client of the UAPs concern for him D. D. Try not to listen to the conversation since it is confidential 162. During the past 30 days an elderly client has exhibited a progressively decreasing appetite, is spending increasing amounts of the daytime hours in bed, and refuses to participate in planned daytime activities. Which action should the nurse take? A. A. Withhold any medications that may cause these side effects B. B. Motivate the client by offering favorite foods as a prize C. C. Ask the family members to visit more often to stimulate the client D. D. Record the findings and report the symptoms to the charge nurse 163. A client is receiving nitroglycerin sublingual tablets for angina. What response should the nurse expect the client to manifest in response to the administration of this drug during an acute anginal episode? A. A. Pulse oximetry within normal limits B. B. Cessation of acute chest pain C. C. Hypertension and headache D. D. Premature ventricular contractions (PVC) 164. After a client returns from Hemodialysis, the nurse measures the client's weight and notes a 3-pound weight loss from the pre-dialysis weight. The client reports feeling weak and fatigued. What action should the nurse take next? A. A. Measure the client's blood pressure B. B. Auscultate the client's breath sounds C. C. Observe the client's legs for edema D. D. Determine the client's blood glucose 165. When providing oral care to an unconscious client who is a mouth breather and does not swallow, which action is most important for the nurse to implement? A. A. Use an oral suction catheter in the buccal cavity B. B. Inspect the oral cavity using gloves fingers C. C. Perform oral cleansing with a sponge toothette D. D. Apply a petroleum based lubricant to the client's lips 166. Wrist restrains were applied to a client who was severely agitated and disoriented. In monitoring the client, who is now asleep, which finding should be reported to the charge nurse? A. A. Respiratory rate decreases from 22 to 16 per minute B. B. Radial pulse volume decreases from +3 to +1= C. C. Blood pressure decreases from 130/84 to 120/76 D. D. Apical pulse rate decreases from 94-84 per minute 167. The nurse is providing wound care for a client with a stage III pressure ulcer on the left heel. To achieve the goal, "An increase in granulation tissue will develop within 2 weeks," which intervention should the nurse implement? A. A. Remove heel protector every two hours B. B. Irrigate wound with sterile normal saline C. C. Replace dry sterile dressings as needed D. D. Apply heat for 15 minutes three times daily 168. A client's chief complaint is being able to swallow only small bites of solid food and liquid's for the last 3 months. The nurse should assess the client for what additional information? A. A. History of alcohol and tobacco use B. B. Average daily consumption of hot beverages C. C. Past traumatic injury to the neck D. D. Daily dietary roughage intake 169. The care plan for a male client with amyotrophic lateral sclerosis includes the Nursing diagnosis, "Decisional conflict related to concerns about mechanical ventilation." When assigned to care for this client, what intervention should the nurse implement based on this diagnosis ? A. A. Provide an opportunity for the client to meet with survivors of the disease who have undergone mechanical ventilation B. B. Remind the client that a mechanical ventilator is usually only needed for a short period of time C. C. Ask the hospice nurse to visit with the client to discuss his options for care if he chooses not to undergo mechanical ventilation D. D. Encourage the client to discuss his feelings and concerns related to the use of mechanical ventilation 170. What is the function of neutrophils? A. A. Heparin secretion B. B. Transport oxygen C. C. Phagocytotic action D. D. Antibody formation 171. Which membrane lines the abdominal cavity A. A. Perineum B. B. Pericardium C. C. Pleura D. D. Peritoneum 172. A man who was brought to the psychiatric hospital by the sheriff because he was hallucinating and stumbling on a downtown street, refuses to wait for a psychiatric evaluation. Which action should the nurse take? A. A. Tell the man when the elevator will see him B. B. Alert the staff to monitor exits to prevent escape C. C. Warn the client that he is likely to have a seizure D. D. Offer a hot meal a clean bed and a sleeping pill 173. The nurse is assessing care for residents on a 12-bed unit in an extended care facility. The staff consists of 1 unlicensed assistive personnel (UAP) and 1 certified medication aide. Which task should the nurse perform? A. A. Ambulate the client who has left hemiplegia and uses a cane B. B. Administer medications and formula to a client with a gastronomy tube C. C. Change a hydrocolloid dressing for a client with a stage II pressure ulcer D. D. Provide self-catheterization equipment for a client with paraplegia 174. The nurse is reviewing the discharge medication instructions with a client for disulfiram 10mg (Antabuse). Which instruction should the PN reinforce with the client? A. A. Avoid all sources of alcohol while taking this drug including cough syrups B. B. The medication should be taken at the same time each day C. C. Stop the drug if nausea, vomiting and/or prostration occur D. D. Have weekly blood tests to determine therapeutic drug levels and serum sodium 175. The nurse is preparing a client for a bone marrow aspiration. Which erythropoietic site is most likely to be used to obtain the specimen? A. A. Vertebrae B. B. Ribs C. C. Cranial bones D. D. Iliac crest 176. A male client admitted the morning of his scheduled surgery tells the nurse that he drank a glass of water during the night. What intervention will the nurse implement first? A. A. Auscultate the client for bowel sounds and ability to urinate B. B. Determine the amount of water and exact time it was taken C. C. Notify the healthcare provider of the client's fluid intake D. D. Reassure the client that a small amount of water is not harmful 177. The nurse is providing care for a client receiving an intravenous antibiotic to treat an infection. Which assessment findings require the most immediate action by the RN? A. A. Warm skin with elastic turgor B. B. Dry mouth with thirst C. C. Low grade fever with diaphoresis D. D. Hives with pruritus 178. The nurse should perform oral suctioning for a client with what problem? A. A. Atelactasis B. B. Dysphasia C. C. Gastric reflux D. D. Dysphagia 179. An elderly client at an adult daycare center with Type2 Diabetes Mellitus becomes unresponsive verbally and then tells the nurse, "I just don't feel right" Which initial action should the nurse take? A. A. Assess temperature B. B. Evaluate deep tendon reflexes C. C. Give 4 ounces of apple juice D. D. Administer glucagon 0.5mg IM 180. A 75-year-old male client with Alzheimer's Disease (AD) is admitted to an extended care facility. What intervention should the nurse include into his client's Nursing care plan? A. A. Describe the activities available to the residents and encourage him to choose the ones he prefers B. B. Introduce the client to the Nursing staff and the residents as soon as possible C. C. Plan to have the same Nursing staf f provide care for the client whenever possible D. D. Encourage the client to remain on the unit for 3 weeks until he is oriented to his new surroundings 181. A newborn infant with a tracheoesophageal repair is receiving Gastrostomy (GT) feedings postoperatively. What intervention should the nurse implement during the GT feedings? A. A. Offer a pacifier during the feedings to satiate the sucking reflex associated with feedings B. B. Flush the GT with 50mL of water and clamp the GT to prevent leakage C. C. Place the infant in the right lateral position to facilitate gastric emptying D. D. Burp the infant after each 10mL of formula administration and re-feed any volume that is spit up 182. Which intervention is within the scope of practice for a nurse? A. A. Demonstrating deep breathing and coughing to postoperative client B. B. Teaching the use of glucometer to a newly diagnosed diabetic client C. C. Presenting support options that are available to those with cancer D. D. Discharge teaching about newly prescribed medications 183. The nurse is preparing a client for a mammogram. What instructions should the nurse provide the client? A. A. Do not exercise the upper body on the day of the procedure B. B. Avoid taking aspirin for one week prior to the procedure C. C. Avoid eating or drinking 6 hours prior to the procedure D. D. Do not use underarm deodorant on the day of the procedure 184. An older client is transferred to the rehabilitation unit with the diagnosis of Cerebrovascular Accident (CVA) with left sided hemiplegia. The nurse addresses the client from the right side, and the client points to the left leg and states, "There is a leg in my bed!" What is the best response by the nurse? A. A. "Your stroke has impaired your ability to recognize your paralyzed leg." B. B. "Look at your legs and you will see that they both belong to you." C. C. "Please explain to me what you thing happened to your leg." D. D. "I know you think there is an extra leg in your bed, but I do not see it." 185. Which technique should the nurse use to give a Z-track intramuscular injection? A. A. Ensure that no air is present in the syringe B. B. Inject the medication into the dorsal gluteal site C. C. Select a 22-gauge, 1 inch needle for injection D. D. Massage the site for 2 minutes after the injection 186. The nurse observe that the IV catheter is no longer in a client's arm. It is on the bed, and the sheets are moist with IV fluid. The client is disoriented and states he does not remember pulling the catheter out. How should the nurse document this situations? A. A. Client does not remember pulling out the IV B. B. IV catheter found lying on bed sheets C. C. IV catheter pulled out by disoriented client D. D. IV discontinued and wet sheets changed 187. The nurse identifies several findings in an older female who is on prolonged bed rest. Which finding requires prompt action by the nurse? A. A. Heart rate increases of 10 beats per minute B. B. Bowel movements decrease to 1 every third day C. C. Urinary output decreases of 250mL in the last 24 hours D. D. D. Systolic blood pressure decrease of 10mmHg 188. A nurse sees a colleague taking drugs from the hospital unit. What action should the nurse take? A. A. Report the incident to the person in charge of the unit or Nursing supervisor B. B. Notify the hospital security staff to retrieve the drugs from the colleague C. C. Report the colleague to the peer review committee of the hospital D. D. Confront the colleague and tell him/her to take the drugs back to the unit 189. Which term describes 2 or more tissues that compose a structure and perform a specific function? A. A. Elastic tissue B. B. Organ C. C. System D. D. Serous membrane 190. How many mL should the nurse document when calculating a client's 8-hour fluid intake? (Enter the numeric value only.) 0730 - 4 ounces of orange juice, hardboiled egg, and toast 1130 - 1/2 cup of soup, one half sandwich, and 1/2 cup of apple juice 1300 - vomitus of 100 mL 1400 - voided 250 ml and consumed one 12-ounce can of soft drink (type your answer in the box below) 1oz = 30mL; so 4oz of orange juice X 30mL = 120mL of orange juice Then 1 cup = 240; so ½ cup is 120mL of soup and ½ cup of apple juice is 120mL of apple juice = 240mL total vomitus is output, not intake, so ignore voided is output, not intake, so ignore 1 oz = 30mL; so 12oz is 12oz X 30mL = 360mL add them all; 120mL + 240mL + 360mL = 720mL 720 191. A male client is receiving ferrous sulfate (iron), docusate sodium (Colace) and codeine. He reports that his last bowel movement was 3 days ago. During medication administration, which action should the nurse implement? A. A. Offer the client a full glass of water B. B. Give medications 2 hours apart C. C. Provide a snack with the medications D. D. Administer only the docusate sodium 192. The nurse is caring for a primagravida 5 hours after a vaginal delivery. Which finding should the nurse report immediately to the charge nurse? A. A. Pulse rate of 90 beats/minute B. B. Rubor lochia saturating 3 perineal pads per hour C. C. Complaints of perineal pain D. D. Firm fundus between umbilicus and the symphysis pubis 193. A client with recurrent urinary tract infections (UTI) is being discharged. What instruction is appropriate for the nurse to include in the discharge teaching plan? A. A. Drink 3 quarts of water daily B. B. Avoid swimming in public pools C. C. Avoid intercourse until all antibiotics have been taken D. D. Drink 3, 6-ounce cans of cranberry juice daily 194. Which criterion is best for the nurse to use when evaluating a client's response to an analgesic that was administered for postoperative pain? A. A. Amount of medication required to relieve pain B. B. Activity without guarding or grimacing C. C. Objective parameters of blood pressure and respirations D. D. Subjective score on a 1 to 10 pain scale 195. A client is diagnosed with Pericarditis after a Myocardial Infarction (MI) and asks the nurse, "Why did this happen?" What explanation should the nurse offer? A. A. The sac surrounding the heart has become inflamed from the cells damaged by the heart attack B. B. The space around your heart is filling with fluid and your healthcare provider will have to explain the treatment C. C. The heart cells have been infiltrated by organisms and a secondary autoimmune reaction has occurred D. D. This is an infection of the lining of the heart caused by bacteria entering through your gums 196. In describing the "at risk" individual for developing Breast Cancer, the nurse should recognize that which client is at the highest risk? The woman who is... A. A. a 40-year-old African American with Hypertension (HTN) B. B. a 35-year-old with trauma to the breast C. C. a 32-year-old whose mother had breast cancer D. D. a 50-year-old Caucasian who has never had a mammogram 197. What technique should the nurse use to administer a medicated ophthalmic ointment? A. A. Massage the lashes with the excess ointment that is squeezed out when shutting the lids B. B. Place a thin ribbon of ointment into the lower conjunctival sac from the inner to outer canthus C. C. Pull both upper and lower lids apart to drop the ointment onto the anterior surface of the eye D. D. Wear gloves when placing the tip of the ointment tube in the center of the lower lid 198. A client is using an incentive spirometer on the first postoperative day after an inguinal Herniorrghaphy. The nurse should re-teach the proper use of the spirometer when the client demonstrates what action? A. A. Using a tight seal around the mouth piece B. B. Exhaling slowly after two seconds C. C. Blowing forcefully into the mouthpiece D. D. Sitting upright during treatment 199. An 8-year-old recovering from a Celiac Crisis requests a bowl of cereal for breakfast. Which cereal should the nurse provide? A. A. Corn flakes B. B. Granola C. C. Oatmeal D. D. Wheat puffs E. E. Rice 200. The nurse assumes care of a client who was admitted earlier in the day for a scheduled Hysterectomy in the morning. Which recorded assessment data obtained by the admitting registered nurse is objective? (Select all that apply). A. A. Anemia B. B. Menorrhagia C. C. Tiredness D. D. Orthostatic hypotension E. E. Fear F. F. Nervousness 201. The nurse empties a large amount of serous drainage from a postoperative client's Hemovac drain. In what order should the nurse implement these procedures? (Place the first action on top and the last action on the bottom.) A. Compress drain... close drain... discard drain... document 202. The nurse should recommend that males over the age of 45 obtain which test to screen for prostatic cancer? A. A. Prostate-specific antigen (PSA) B. B. Alpha-fetoprotein radio immunoassay (AFP) C. C. Ultrasound of the scrotum D. D. Serum testosterone level 203. The nurse is giving medications to a client who was admitted to the hospital with a diagnosis of Diabetes Mellitus Type II. After checking the finger stick glucose at 1630dL, what dose of insulin should the nurse administer? (enter the numeric value only) (Click on each chart tab for additional information. Please be sure to scroll to the bottom- right corner of each tab to view all information contained in the client's medical record.) 8 204. A client is receiving 0.5 grams of a prescription medication that is dispensed as 500 mg/5mL. How many ml should the PN administer? (enter the numeric value only. If rounding is required, round to the nearest tenth.) 5 205. The nurse is receiving a client following an emergency Cesarean Section (C-Section). Which information is most important for the nurse to obtain? A. A. Blood pressure and pulse rate B. B. Gravida and parity C. C. Medications received during labor D. D. Temperature and respiratory rate 206. The nurse is preparing to insert an indwelling catheter for an 89- year-old client who has severe contractures of both lower extremities. The client cries in pain when positioned supine while the nurse attempts to abduct the hips to visualize the perineum. What action should the nurse take? A. A. Report to the charge nurse that the client cannot cooperate for the insertion B. B. Recruit two UAPs to hold the legs apart while the catheter is inserted C. C. Position laterally for posterior access in visualizing the meatus for insertion D. D. Pre-medicate the client with a narcotic analgesic to relax the skeletal muscles 207. An elderly client in the early postoperative period requires close monitoring due to aging and multisystem changes. The nurse monitors respirations and auscultates breath sounds frequently. What other intervention should the nurse implement related to the client's decreased vital capacity? A. A. Evaluate pulse oxygen saturation B. B. Allow extra education time C. C. Encourage high protein supplements D. D. Monitor intake and output 208. The nurse can also refer to the external ear as what other known name... A. A. Pinna B. B. Malleus C. C. Incus D. D. Cochlea 209. During immediate postoperative period, which condition has the highest priority when planning Nursing care? A. A. Infection B. B. Respiratory obstruction C. C. Dehydration D. D. Cardiac arrest 210. The nurse is providing instructions to the unlicensed assistive personnel (UAP) preparing to instruction is most important for the nurse to emphasize? A. A. Keep the head of the bed raised while the tube feeding is infusing B. B. Report any drainage observed around the GT insertion site C. C. Raise the entire bed while bathing the client to reduce back strain D. D. Use plenty of pillows to position the client on the side after bathing 211. A client is admitted to the rehabilitation unit after a Thrombotic Cerebrovascular Accident (CVA) with Right Hemiplegia and expressive aphasia. What intervention should the nurse implement to communicate with the client? A. A. Picture communication board B. B. Request a family member to interpret C. C. Electronic larynx device D. D. Dysphagia precautions 212. The nurse is reviewing instructions for the use of pilocarpine eye drops with a client who has Glaucoma. The client states, "I should have these drops to anesthetize my eye if I experience pain" What action should the nurse implement? A. A. Explain to the client the eye drops do provide pain relief, but do not anesthetize the eyes B. B. Reassure the client that the drops will not be needed often since eye pain in glaucoma is not common C. C. Re-teach the client about the action of the eye drops to decrease pressure in the eye D. D. Document in the chart that the client understands the action and use the eye drops 213. A client is complaining of muscle fatigue in the lower extremities. What is the physiological cause of muscle fatigue? A. A. The depletion of glycogen and energy stores B. B. Electrical stimulus failure at the neuromuscular junction C. C. Calcium concentration decrease in the muscle sarcomere D. D. Hyperoxygenation of the muscle fiber 214. A client asks the nurse to explain the location of the prostate gland. What is the best response? A. A. Close the rectal wall the prostate gland sits behind the symphysis pubis extending around the beginning of the urethra B. B. At the bottom of the scrotal sac, the prostate gland rests beneath the testes, held in place by the spermatic fascia C. C. Attach to the front and sides of the pubic arch, the prostate is a mess of cavernous tissue held together by fibrous tissue D. D. Located at the lateral edge of the posterior segment of the testes, the prostate creates a bulge continuous with the vas deferens 215. A female client is being prepared for a speculum exam. In which position should the nurse place the client? A. A. Left Sims B. B. Semi-Fowler's C. C. Lithotomy D. D. Trendelenburg 216. The nurse is caring for an elderly client who has suddenly become confused after 2 days of vomiting and diarrhea. What laboratory result should the nurse report first to the RN? A. A. Serum potassium 6mEq/L, serum sodium 126mnEq/L, and serum chloride 115mEq/L B. B. Glucose tolerance results fasting 80 mg/dL, 1hr: 110mg/dL 2hr: 120 mg/dL, 3hr: 90 mg/dL C. C. Negative Hepatitis B Surface Antigen, serum total biilirubin 0. 1 mg/dL D. D. Troponin l < 0.1ng/mL and creatinine kinase MB (CK-MB) 2% of total 10 milliunits/L 217. While providing oral care for a client who is unconscious, the nurse positions the client laterally and uses a basin to collect secretions. Which intervention is best for the nurse to implement? A. A. Swab the oral cavity with a washcloth B. B. Use oral swabs with normal saline C. C. Provide a Yankauer tip for oral suction D. D. Support the head with a small pillow 218. The nurse is caring for a mother who is bottle-feeding and develops breast engorgement. Which intervention is most effective in reducing breast engorgement? A. A. Wearing a tight-fitting bra B. B. Applying hot packs to the breasts C. C. Expressing milk from the breast by hand D. D. Exposing the breasts to air 219. A 6-month old male with Bronchiolitis is admitted to the hospital. In monitoring the respiratory status of this child, which symptom indicates the nurse that he is experiencing Respiratory Distress? A. A. Respiratory of 62 breaths/minute B. B. Abdominal breathing C. C. A high-pitched cry D. D. Dry flushed skin 220. During vital sign assessment of a client, the nurse counts the left radial pulse at 88, and the pulse oximeter clipped to a finger on the left hand records a pulse rate of 68 with an oxygen saturation of 95%. What is the best initial action by the nurse? A. A. Count the right radial pulse rate B. B. Reposition the oximeter clip C. C. Document a pulse deficit D. D. Count the apical pulse rate 221. Which client should the nurse assess first? A. A. A young female client who reports that she is afraid of her roommate who is psychotic B. B. An older client who is asking for a priest to offer Last Rites C. C. A female client who is anxious about being discharged because she has no assistance at home D. D. A client who is ambulating with partial weight-bearing after a total hip replacement [Show More]

Last updated: 1 year ago

Preview 1 out of 74 pages

Reviews( 0 )

$16.50

Add to cart

Instant download

Can't find what you want? Try our AI powered Search

OR

GET ASSIGNMENT HELP
15
0

Document information


Connected school, study & course


About the document


Uploaded On

Mar 06, 2023

Number of pages

74

Written in

Seller


seller-icon
Nolan19

Member since 2 years

10 Documents Sold


Additional information

This document has been written for:

Uploaded

Mar 06, 2023

Downloads

 0

Views

 15

Document Keyword Tags

Recommended For You


$16.50
What is Browsegrades

In Browsegrades, a student can earn by offering help to other student. Students can help other students with materials by upploading their notes and earn money.

We are here to help

We're available through e-mail, Twitter, Facebook, and live chat.
 FAQ
 Questions? Leave a message!

Follow us on
 Twitter

Copyright © Browsegrades · High quality services·